2023 UKFPO SJT Practice Paper Flashcards

1
Q
  1. You have been working in the Acute Admissions Unit* (AAU) for the past four months and a patient has been admitted for severe abdominal pain. This is the third time that the patient has been admitted for the same condition and you have conducted all of her previous assessments. You go to assess the patient and when she sees you, she says, “No, I do not want you as my doctor again. You can’t find out what is wrong with me. Please go and get me an experienced doctor.”
    Rate the appropriateness of the following responses to say in this situation. Assume they are all said in a polite way. (1= Very appropriate; 4= Inappropriate).
    A. “Please be assured that I will try my best to find out the cause of your pain.”
    B. “I’m here to help you, please don’t speak to me in that way.”
    C. “Let me see if a senior doctor is available to assess you instead.”
    D. “I’m sorry you feel that way, but it is not my fault you are still experiencing pain.”
    E. “I can see if a senior doctor is available, however, you might end up having to wait a while to be seen.”
A

A) “Please be assured that I will try my best to find out the cause of your pain.” Correct Key: 1 - Very appropriate
Rationale: This response is very appropriate because as a doctor you have a responsibility to work with your patient to address their concerns (General Medical Council [GMC] good medical practice’s Communication, Partnership and Teamwork domain). Therefore, prioritising trying to find the cause of the patient’s pain is the appropriate thing to aim to do.
B) “I’m here to help you, please don’t speak to me in that way.” Correct Key: 3 - Somewhat inappropriate
Rationale: This response is somewhat inappropriate because whilst communication is integral to establishing a good working relationship with your patient (GMC good medical practice’s Communication, Partnership and Teamwork domain) and it is important to establish boundaries to be able to develop and/or maintain a respectful dialogue between both parties, this response may be interpreted by the patient as confrontational and unprofessional.
C) “Let me see if a senior doctor is available to assess you instead.” Correct Key: 2 - Somewhat appropriate
Rationale: This response is somewhat appropriate because it is your responsibility as a doctor to respond to the patient’s concern (GMC good medical practice’s Communication, Partnership and Teamwork domain), in this case, their concern is not wanting to be treated by you. The four pillars of medical ethics include autonomy, which is the patient’s right to make decisions about their health: This response respects the patient’s right to choose who treats them. According to the NHS Patient’s Charter, a patient also has the right to seek a second opinion on their medical condition. Therefore, this response is acknowledging the patient’s right to have another doctor assess their condition instead of you (GMC good medical practice’s Knowledge, Skills and Performance domain), however you haven’t reassured the patient that she will be assessed by another doctor as requested.
D) “I’m sorry you feel that way, but it is not my fault you are still experiencing pain.”
Correct Key: 4 - Inappropriate
Rationale: This response is an inappropriate response as you have a duty as a doctor to treat patients with respect and dignity (GMC good medical practice’s Communication, Partnership and Teamwork domain). Although the patient has refused treatment from you, it is important that you maintain your professional values and be respectful in your response back to the patient (GMC good medical practice’s Maintaining Trust domain) to maintain the patient’s trust in you as a professional and their faith in the profession.
E) “I can see if a senior doctor is available, however, you might end up having to wait a while to be seen.”
Correct Key: 2 - Somewhat appropriate
Rationale: This response is somewhat appropriate because one of your responsibilities as a doctor is to ensure the provision of sufficient information about treatment options so that patients can make an informed choice about their medical care, including the consequences of refusing treatment (GMC good medical practice’s Knowledge, Skills and Performance domain and the Communication, Partnership and Teamwork domain).

How well did you know this?
1
Not at all
2
3
4
5
Perfectly
2
Q
  1. Part A You have volunteered to be a facilitator on a clinical exam for medical students. A fourth-year medical student, Sarah, approaches you while you are walking out of work. Sarah knows that you are one of the facilitators for the clinical exam and asks you what type of scenarios will be covered in the clinical exam. You have worked with Sarah closely over the last few months.
    Rate the appropriateness of the following actions in response to this situation. (1= Very appropriate; 4= Inappropriate).
    A. Provide Sarah with some publicly available information about medical school examination
    B. Explain to Sarah that it would be unfair on other candidates if only she knew the scenarios before the clinical exam
    C. Reassure Sarah that she should not need to know the scenarios before the clinical exam in order to do well
A

A) Provide Sarah with some publicly available information about medical school examinations Correct Key: 1 - Very appropriate
Rationale: This is a very appropriate action because it is part of a doctor’s responsibility to develop themselves and to support others to develop (General Medical Council [GMC] good medical practice’s Communication, Partnership and Teamwork domain). Therefore, it is appropriate to provide Sarah with some guidance without giving her unfair advantage over other students and compromising your professional values such as, honesty, integrity and probity (GMC good medical practice’s Maintaining Trust domain).
B) Explain to Sarah that it would be unfair on other candidates if only she knew the scenarios before the clinical exam
Correct Key: 1 - Very appropriate
Rationale: This is a very appropriate action because as a doctor, it is important to maintain positive personal and professional values such as honesty, integrity and probity. Sharing the scenarios with Sarah would amount to you being dishonest and complicit in Sarah cheating in a clinical exam (GMC good medical practice’s Maintaining Trust domain). Apart from being morally wrong, giving Sarah an unfair advantage over her peers by cheating has other implications relating to clinical competence and patient safety as a result of Sarah not learning, revising, consolidating her knowledge and skills to be truly proficient (GMC good medical practice’s Knowledge, Skills and Performance domain).
C) Reassure Sarah that she should not need to know the scenarios before the clinical exam in order to do well
Correct Key: 2 (somewhat appropriate)
Rationale: This action is somewhat appropriate as Sarah’s request may be a result of fear or anxiety relating to her clinical exams. Reassuring Sarah about her potential and ability may be sufficient to enable her to reflect on her actions in requesting access to the scenarios beforehand. However, this action does not address the underlying issues and concerns about honesty, integrity and probity that her actions raise (GMC good medical practice’s Maintaining Trust domain).

How well did you know this?
1
Not at all
2
3
4
5
Perfectly
3
Q
  1. Part A (for reference): You have volunteered to be a facilitator on a clinical exam for medical students. A fourth-year medical student, Sarah, approaches you while you are walking out of work. Sarah knows that you are one of the facilitators for the clinical exam and asks you what type of scenarios will be covered in the clinical exam. You have worked with Sarah closely over the last few months.
    Part B: The following day, you see an FY1 colleague, James, talking with Sarah over lunch. James is also a facilitator for the clinical exam. You can hear that James is providing Sarah with specific information about the types of scenarios which will be covered on the clinical exam. As James leaves the room, you walk over to Sarah and when she sees you, looks very guilty.
    Rate the appropriateness of the following actions in response to this situation. (1= Very appropriate; 4= Inappropriate).
    A. Ask Sarah why she has chosen to also ask James for information
    B. Tell Sarah that you will need to inform her educational supervisor* about the situation
    C. Encourage Sarah to declare to the medical school that she has been told information about the clinical exam
    D. Explore with Sarah why she feels she needs to know the information
A

A) Ask Sarah why she has chosen to also ask James for information Correct Key: 1 - very appropriate
Rationale: This action is very appropriate as it is important to question and challenge colleagues’ behaviour and professionalism when appropriate. Being privy to Sarah’s desire to know the clinical scenarios in advance of the exam from your interaction with her the day before, it is your responsibility to ascertain if she has made the same request to your colleague. If she has, this is questionable conduct which needs to be addressed (GMC good medical practice’s Maintaining Trust domain).
B) Tell Sarah that you will need to inform her educational supervisor* about the situation Correct Key: 2 - Somewhat appropriate
Rationale: This is a somewhat appropriate action as it is part of your responsibility to raise concern about conduct and performance where indicated (GMC good medical practice’s Safety and Quality domain, Maintaining Trust domain). Informing Sarah of your intention to do so though, is optional.
C) Encourage Sarah to declare to the medical school that she has been told information about the clinical exam
Correct Key: 1 - Very appropriate
Rationale: This is a very appropriate action to take as it is important that Sarah takes responsibility for her actions, including when she makes a mistake or error of judgement. Encouraging Sarah to disclose this to the medical school is essential in terms of her demonstrating positive professional values as required, such as honesty, integrity and probity (GMC good medical practice’s Maintaining Trust domain).
D) Explore with Sarah why she feels she needs to know the information
Correct Key: 1 - Very appropriate
Rationale: This is a very appropriate action to take because supporting others to develop is part of your responsibility as a doctor (General Medical Council [GMC] good medical practice’s Communication, Partnership and Teamwork domain). Supporting Sarah to explore the reasons behind her action may be useful in helping her develop positive coping strategies and assessment management strategies to help her cope with clinical exams/assessments in the future, as well as other pressures she may come across as part of her ongoing development.

How well did you know this?
1
Not at all
2
3
4
5
Perfectly
4
Q
  1. You are working in the Emergency Department, and a 19 year old patient presents alone, after being elbowed in the eye while playing football. When you inform the patient that she has broken her eye socket and may need to have surgery, she becomes very upset. She tells you that she does not want to have surgery and that she wants to go home.
    Rate the appropriateness of the following responses to say in this situation. Assume they are all said in a polite way. (1= Very appropriate; 4= Inappropriate).
    A. “It is up to you whether or not you go home.”
    B. “Is there is a particular reason why you do not want to have surgery?”
    C. “Do you have any family members or friends who can come to support you?”
A

A) “It is up to you whether or not you go home.” Correct Key: 4 - Inappropriate
Rationale: This is an inappropriate response because as a doctor, you most prioritise patient safety (General Medical Council [GMC] good medical practice’s Safety and Quality domain) as it is in the best interest of the patient (Four Pillars of Medical Ethics – Beneficence and Non-maleficence). It is also important that you address the patient’s concern(s) which is making her upset (GMC good medical practice’s Communication, Partnership and Teamwork domain) and this response fails to do so.
B) “Is there is a particular reason why you do not want to have surgery?” Correct Key: 1 - Very appropriate
Rationale: This response is very appropriate because as a doctor it is your responsibility to listen to your patient and to address their concerns (GMC good medical practice’s Communication, Partnership and Teamwork domain). Establishing the reason(s) for this patient getting upset may be the key to establishing a positive working relationship, enabling her to engage with treatment which is in her best interest (Four Pillars of Medical Ethics – Beneficence and Non-maleficence).
C) “Do you have any family members or friends who can come to support you?”
Correct Key: 1 - Very appropriate
Rationale: This is a very appropriate response because as a doctor, your duty of care to your patient includes responding to your patient’s concerns as well as supporting your patients to care for themselves (GMC good medical practice’s Communication, Partnership and Teamwork domain). Therefore, finding out what support the patient needs and has will enable you to be able to do so in a meaningful way that is also in the best interest of the patient (Four Pillars of Medical Ethics – Beneficence and Non-maleficence).

How well did you know this?
1
Not at all
2
3
4
5
Perfectly
5
Q
  1. Part A You are working on a paediatric ward and a nurse, who works in a different department, approaches you to asks about his 6 year old niece who is a patient on your ward. He asks what is wrong with her and what treatment she is receiving. You are aware that the nurse is not the patient’s legal guardian. You have a good relationship with the nurse and you can see that he is clearly anxious about his niece.
    Rate the appropriateness of the following responses to say in this situation. Assume they are all said in a polite way. (1= Very appropriate; 4= Inappropriate).
    A. “You know that this is confidential information that I cannot discuss with you.”
    B. “You will need to speak with your niece’s parents directly about this.”
    C. “Try not to worry. I’m sure everything is going to be fine.”
    D. “It is inappropriate for you to ask this of me so please do not ask me again.”
A

A) “You know that this is confidential information that I cannot discuss with you.” Correct Key: 2 - Somewhat appropriate
Rationale: This response is somewhat appropriate because although the nurse is your colleague and your patient’s relative, they do not have parental responsibility for your patient and therefore have no legal right to be informed about your patient’s private health information. It is important that you respect your patient’s right to confidentiality (General Medical Council [GMC] good medical practice’s Communication, Partnership and Teamwork domain) and that you treat the patient’s information that has been entrusted to you with respect (GMC good medical practice’s Communication, Partnership and Teamwork domain and Maintaining Trust domain). However, the nurse is clearly anxious about his niece and he may interpret your response as being unsympathetic.
B) “You will need to speak with your niece’s parents directly about this.” Correct Key: 1 - Very appropriate
Rationale: This response is very appropriate as it respects your patient’s right to confidentiality (GMC good medical practice’s Communication, Partnership and Teamwork domain) by referring your colleague to his relatives who have parental rights over your patient. This means that you are not breaching the patient’s confidentiality by disclosing their private information to a colleague.
C) “Try not to worry. I’m sure everything is going to be fine.” Correct Key: 4 - Inappropriate
Rationale: This response is inappropriate because although you are not disclosing specific details of your patient’s condition or status to your colleague, you are breaching your patient’s right to confidentiality by alluding/indirectly referring to their current medical status in your response (GMC good medical practice’s Communication, Partnership and Teamwork domain). The response is also providing assurances that cannot be guaranteed, which may prove to be problematic in the future if the patient’s condition deteriorates and everything is not as predicted.
D) “It is inappropriate for you to ask this of me so please do not ask me again.”
Correct Key: 3 - Somewhat inappropriate
Rationale: This response is somewhat inappropriate because although your colleague should not be asking you to disclose confidential information, it is important that you recognise the motive behind his request is his concern for his niece and respond in a compassionate manner while preserving your patient’s confidentiality (GMC good medical practice’s Communication, Partnership and Teamwork domain).

How well did you know this?
1
Not at all
2
3
4
5
Perfectly
6
Q
  1. Part A (for reference): You are working on a paediatric ward and a nurse, who works in a different department, approaches you to asks about his 6 year old niece who is a patient on your ward. He asks what is wrong with her and what treatment she is receiving. You are aware that the nurse is not the patient’s legal guardian. You have a good relationship with the nurse and you can see that he is clearly anxious about his niece.
    Part B: The following day, you arrive on the paediatric ward and notice the nurse reading a set of patient notes. When he sees you approaching, he looks guilty and immediately puts the notes back and walks away. You do not know which set of notes he was reading but suspect that he may have been reading his niece’s notes.
    Rate the appropriateness of the following actions in response to this situation. (1= Very appropriate; 4= Inappropriate).
    A. Ask the nurse in charge for advice about how to respond
    B. Tell the patient’s parents that you suspect the nurse was reading their daughter’s notes
    C. Ask the nurse why he was reading a set of patient notes for a ward he does not work on
A

A) Ask the nurse in charge for advice about how to respond Correct Key: 1 - Very appropriate
Rationale: This is a very appropriate action to take because it is your responsibility to preserve the dignity and confidentiality of your patient as well as to raise concern when you witness inappropriate conduct (GMC good medical practice’s Communication, Partnership and Teamwork domain). However, as you are not certain about what notes your colleague may have been reading, it is important to bring what you have witnessed to the attention of the nurse in charge for advice about how to respond to the situation.
B) Tell the patient’s parents that you suspect the nurse was reading their daughter’s notes Correct Key: 3 - Somewhat inappropriate
Rationale: This is a somewhat inappropriate action to take because although it is your responsibility to preserve the dignity and confidentiality of your patient as well as to raise concern when you suspect or witness inappropriate conduct (GMC good medical practice’s Communication, Partnership and Teamwork domain), there are mechanisms to raise concern appropriately within the Trust and it is important that you follow those processes.
C) Ask the nurse why he was reading a set of patient notes for a ward he does not work on Correct Key: 1 - Very appropriate
Rationale: This is a very appropriate action to take because it is your responsibility to preserve the dignity and confidentiality of your patient as well as to raise concern when you witness inappropriate conduct (GMC good medical practice’s Communication, Partnership and Teamwork domain), including challenging your colleague directly regarding his behaviour on the ward.

How well did you know this?
1
Not at all
2
3
4
5
Perfectly
7
Q
  1. You have had a particularly busy and challenging day on the ward and are feeling overwhelmed and finding it hard to concentrate. You ask your clinical supervisor*, Cathy, if you can leave two hours early because of how you feel and she says, “you need to toughen up, there are not enough staff on the ward anyway so you can’t leave.”
    Rate the importance of the following considerations in the management of this situation. (1= Very important; 4= Not at all important).
    A. The impact on the rest of the ward staff if you leave work now
    B. The risk to patient safety if you complete your shift feeling like you do
    C. The amount of time you have left until the end of your shift
A

A) The impact on the rest of the ward staff if you leave work now Correct Key: 1 - Very important
Rationale: This is a very important because managing the workload is a collaborative effort and it would be inappropriate, irresponsible and negligent to leave the ward short without adequate staff (General Medical Council [GMC] good medical practice’s Communication, Partnership and Teamwork domain) to maintain safe care (GMC good medical practice’s Safety and Quality domain).
B) The risk to patient safety if you complete your shift feeling like you do Correct Key: 1 - very important
Rationale: This is very important because you must prioritise patient safety and ensure that you work within your own limits (GMC good medical practice’s Knowledge, Skills and Performance domain), including when you are feeling overwhelmed and unable to concentrate (GMC good medical practice’s Safety and Quality domain). In this case, it may be more appropriate to speak to your supervisor for advice about how to best manage the workload and how you are feeling, including the possibility of prioritising the workload with support, taking a break to recuperate, working collaboratively with the rest of the team to ensure that patient care is appropriately covered.
C) The amount of time you have left until the end of your shift Correct Key: 3 - Of minor importance
Rationale: This is of minor importance because patient care and patient safety most be prioritised (GMC good medical practice’s Knowledge, Skills and Performance domain and the Safety and Quality domain) at all times, regardless of how much time you have left until the end of the shift.

How well did you know this?
1
Not at all
2
3
4
5
Perfectly
8
Q
  1. You are working on a paediatric ward and you have prescribed a drug for a patient after being advised to do so by the consultant. The nurse looking after the patient has informed you that she will not give the drug to the patient as she has checked your calculation and believes that it is the wrong dose. You were confident that your calculations were correct. The drug was due to be given half an hour ago, and the patient’s parents are concerned that it is delayed. You know that the timing of the dose is not critical.
    Rate the appropriateness of the following actions in response to this situation. (1= Very appropriate; 4= Inappropriate).
    A. Reassure the parents that the delay in giving the drug will not cause harm to their child
    B. Tell the parents that the delay is because the nurse thinks that your drug calculations are incorrect
    C. Ask the consultant to confirm with yourself and the nurse what dose to give to the patient
    D. Avoid having a conversation with the parents until after you have sorted out the issue with the drug dose
A

A) Reassure the parents that the delay in giving the drug will not cause harm to their child Correct Key: 1 - Very appropriate
Rationale: This action is very appropriate as it is important to ensure that you address and respond to a patient and/or their family’s concerns (General Medical Council [GMC] good medical practice’s Communication, Partnership and Teamwork domain) by reassuring them that the timing of the medication is not critical.
B) Tell the parents that the delay is because the nurse thinks that your drug calculations are incorrect
Correct Key: 4 - Inappropriate
Rationale: This action is inappropriate and unprofessional as this is attempting to blame the nurse for the delay when in essence, the nurse has rightly raised concern about a potential drug error, which is a significant safety issue (GMC good medical practice’s Patient Safety and Quality domain). This response may also result in lack of trust and reputation damage (GMC good medical practice’s Communication, Partnership and Teamwork domain and the Maintaining Trust domain) which would be detrimental to establishing an effective working relationship.
C) Ask the consultant to confirm with yourself and the nurse what dose to give to the patient
Correct Key: 2 - Somewhat appropriate
Rationale: This action is somewhat appropriate because it is your responsibility as a doctor to seek advice from a senior colleague when appropriate (GMC good medical practice’s Knowledge, Skills and Performance domain and the Communication, Partnership and Teamwork domain) to ensure that the care that is being delivered is safe and effective (GMC good medical practice’s Patient Safety and Quality domain). It could be argued that the need to involve the nurse in the discussion with the consultant is unnecessary.
D) Avoid having a conversation with the parents until after you have sorted out the issue with the drug dose
Correct Key: 4 - Inappropriate
Rationale: This action is very inappropriate as it is important to ensure that you address and respond to a patient and/or their family’s concerns (General Medical Council [GMC] good medical practice’s Communication, Partnership and Teamwork domain) in a prompt and timely fashion as to not cause any distress to the patient or their family (Four Pillars of Medical Ethics – Beneficence and Non-Maleficence).

How well did you know this?
1
Not at all
2
3
4
5
Perfectly
9
Q
  1. While you are working on a ward, your consultant walks past you and asks you to order an urgent chest x-ray* for a patient. You attempt to ask him why, but he rushes off to treat another critically ill patient. The patient approaches you and asks for an update. When you tell the patient that you are about to order another chest x-ray, she tells you that she had a chest x-ray yesterday and asks why she needs another one.
    Rate the appropriateness of the following responses to say in this situation. Assume they are all said in a polite way. (1= Very appropriate; 4= Inappropriate).
    A. “The consultant asked for the chest x-ray but I’m not sure why.”
    B. “I’m not sure why you need one if you had one yesterday; I’ll cancel the chest x-ray.”
    C. “I’ll check that the consultant realises that you had a chest x-ray yesterday.”
A

A) “The consultant asked for the chest x-ray but I’m not sure why.” Correct Key: 3 - Somewhat inappropriate
Rationale: This response is somewhat inappropriate because although it is an honest response, it does not instil much confidence in your ability as a doctor that you have not found out the reasons for the chest x-ray request before addressing the patient (General Medical Council [GMC] good medical practice’s Patient Safety and Quality domain and the Knowledge, Skills and Performance domain). It would be more appropriate to reassure the patient by addressing her concerns when you have all the facts you need to do so effectively (GMC good medical practice’s Communication, Partnership and Teamwork domain and the Maintaining Trust domain).
B) “I’m not sure why you need one if you had one yesterday; I’ll cancel the chest x-ray.” Correct Key: 4 - Inappropriate
Rationale: This is an inappropriate response because it is irresponsible and clinically negligent to make a clinical decision without having all the facts to cancel the chest x-ray (GMC good medical practice’s Patient Safety and Quality domain and the Knowledge, Skills and Performance domain). This action is also unethical because it is not in the best interest of the patient as there may be significant health implications from this action (Four Pillars of Medical Ethics – Beneficence and Non-Maleficence).
C) “I’ll check that the consultant realises that you had a chest x-ray yesterday.” Correct Key: 1 - Very appropriate
Rationale: This is a very appropriate response as it is your responsibility as a doctor to ensure that you and the rest of the team have the most up to date and/or relevant information about a patient in order to make decisions about treatment options and to provide the best quality of care (GMC good medical practice’s Patient Safety and Quality domain and the Knowledge, Skills and Performance domain). This action is also in the best interest of the patient as it ensures that the patient is not unnecessarily over exposed to radiation by having another chest x-ray, particularly if it is not required (Four Pillars of Medical Ethics – Beneficence and Non-Maleficence).

How well did you know this?
1
Not at all
2
3
4
5
Perfectly
10
Q
  1. A patient, Helen, was admitted to your ward overnight with abdominal pain and had an ultrasound this morning, which shows that she is pregnant. Her partner visits the ward and sees this written in her notes. You are outside Helen’s room when you overhear her partner saying that he cannot be the baby’s father. You are unsure whether to intervene in case the situation escalates, but his voice gets louder and he beings to shout at Helen.
    Rate the importance of the following considerations in the management of this situation. (1= Very important; 4= Not at all important).
    A. That Helen and her partner are having a private conversation
    B. That Helen’s patient confidentiality is at risk of being compromised
    C. That Helen’s partner may become physically aggressive
    D. That it is not your job to intervene between patients and their families
A

A) That Helen and her partner are having a private conversation Correct Key: 3 - Of minor importance
Rationale: This consideration is of minor importance because although the conversation is private, it is taking place on the ward and has been overheard. What is more important is prioritising the safety and wellbeing of your patient (General Medical Council [GMC] good medical practice’s Patient Safety and Quality domain).
B) That Helen’s patient confidentiality is at risk of being compromised Correct Key: 2 - Important
Rationale: This consideration is important because it is your responsibility as Helen’s doctor to preserve the dignity of your patient and to ensure that their confidential information is kept safe (GMC good medical practice’s Communication, Partnership and Teamwork domain). However, the main priority is the safety and wellbeing of your patient.
C) That Helen’s partner may become physically aggressive Correct Key: 1 - Very important
Rationale: This consideration is very important because the safety and wellbeing of your patient is a priority (GMC good medical practice’s Patient Safety and Quality domain) and ensuring that she is safe while she is vulnerable is essential to maintain her wellbeing as well as that of her baby (Four Pillars of Medical Ethics – Beneficence and Non-Maleficence).
D) That it is not your job to intervene between patients and their families
Correct Key: 3 - Of minor importance
Rationale: This consideration is of minor importance because as a patient under your care, it is your responsibility to prioritise their care and needs (GMC good medical practice’s Patient Safety and Quality domain and the Communication, Partnership and Teamwork domain).

How well did you know this?
1
Not at all
2
3
4
5
Perfectly
11
Q
  1. You have been working at a hospital for the past 3 months. Your consultant, Dr Wong, recently changed the structure of the FY1 rota, which now includes working more consecutive night shifts. Since the rota has changed, you have noticed that you and your colleagues are more tired and team morale appears low. Today, whilst you are on the ward, Dr Wong asks you “How is everyone finding the new rota?”
    Rate the appropriateness of the following responses to say in this situation. Assume they are all said in a polite way. (1= Very appropriate; 4= Inappropriate).
    A. “Was it necessary to change the rota?”
    B. “It might be a good idea to discuss the new rota at a meeting with the whole team.”
    C. “I think it’s OK, but you should speak to the rest of the team to see what they think.”
A

A) “Was it necessary to change the rota?” Correct Key: 4 - Inappropriate
Rationale: This response is inappropriate because it is most probably not the best way to open up a meaningful dialogue with your senior colleague about an issue that is having such a negative and significant impact on the team (General Medical Council [GMC] good medical practice’s Communication, Partnership and Teamwork domain). It would be more appropriate to express concern(s) in a more diplomatic manner in order to encourage your consultant to explore the issue in more detail with you and the rest of the team.
B) “It might be a good idea to discuss the new rota at a meeting with the whole team.” Correct Key: 1 - Very appropriate
Rationale: This response is very appropriate so that concerns can be raised and discussed with the rest of the team (GMC good medical practice’s Communication, Partnership and Teamwork domain). Research indicates that fatigue/tiredness are linked with a higher risk of errors in clinical practice, therefore having the opportunity to express this concern as a group would be important in terms of minimising potential risk to patients (GMC good medical practice’s Patient Safety and Quality domain and the Communication, Partnership and Teamwork domain). It is also your collective responsibility as doctors to flag up any systemic issues that may negatively impact on patients and staff (GMC good medical practice’s Patient Safety and Quality domain and the Communication, Partnership and Teamwork domain).
C) “I think it’s OK, but you should speak to the rest of the team to see what they think.”
Correct Key: 3 - Somewhat inappropriate
Rationale: This response is somewhat inappropriate as it does not reflect your observations or your concerns about the impact of the change in rota on the team. It is important that as a doctor you raise concerns about risk, unsafe practice, or conduct (GMC good medical practice’s Communication, Partnership and Teamwork domain). It is also important that you are honest and act with integrity when providing feedback (GMC good medical practice’s Maintaining Trust domain).

How well did you know this?
1
Not at all
2
3
4
5
Perfectly
12
Q
  1. Part A: You are working on a general medical ward. Over the past week, you have noticed that your FY2 colleague, Eliza, has been more withdrawn than normal. She has also been more easily irritated, which is unlike her typically friendly manner. Earlier today, you noticed that Eliza was very abrupt with one of the student nurses, who looked upset in response. As you enter the staff room to have your lunch, you overhear some of the other nurses complaining about how rude Eliza has been recently. The nurses see you and continue talking.
    Rate the appropriateness of the following actions in response to this situation. (1= Very appropriate; 4= Inappropriate).
    A. Ignore the nurses, leaving the staff room to eat your lunch elsewhere
    B. Find your FY2 colleague to tell her what you heard the nurses saying about her
    C. Speak to your FY2 colleague to find out if there is a reason for the recent change in her behaviour
    D. Advise your FY2 colleague that you have observed a change in behaviour that is having a negative impact at work
A

A) Ignore the nurses, leaving the staff room to eat your lunch elsewhere Correct Key: 3 - Somewhat inappropriate
Rationale: This action is somewhat inappropriate because as a doctor, you must treat colleagues fairly and with respect (General Medical Council [GMC] good medical practice’s Communication, Partnership and Teamwork domain). Ignoring the nurses and leaving the staff room to eat elsewhere does not reflect a positive work ethic or teamwork and may be detrimental to your working relationship with other members of the team.
B) Find your FY2 colleague to tell her what you heard the nurses saying about her Correct Key: 4 - Inappropriate
Rationale: This action is inappropriate because it is unprofessional and not in the best interest of fostering positive working relationships between the different member of staff (GMC good medical practice’s Communication, Partnership and Teamwork domain). This approach may also impact negatively on your FY2 colleague’s confidence, further isolating her from the rest of the team and worsening the already strained working relationships.
C) Speak to your FY2 colleague to find out if there is a reason for the recent change in her behaviour
Correct Key: 1 - Very appropriate
Rationale: This action is very appropriate because it is your responsibility as a doctor and as a colleague to raise concern about your colleague’s change in behaviour. It is also important to do this in a confidential and supportive manner in order to ascertain how to best support your colleague though the challenges she may be facing (GMC good medical practice’s Communication, Partnership and Teamwork domain and the Maintaining Trust domain).
D) Advise your FY2 colleague that you have observed a change in behaviour that is having a negative impact at work
Correct Key: 2 - Somewhat appropriate
Rationale: This action is somewhat appropriate because it is your responsibility as a doctor and as a colleague to raise concern about your colleague’s change in behaviour. Although it is the FY2 colleague’s responsibility as a doctor to be aware of the impact of her behaviour/change in behaviour on others (GMC good medical practice’s Communication, Partnership and Teamwork domain), it is important that you support colleagues who may have problems with their performance or health and that you are honest when giving feedback to your colleague about your observations (GMC good medical practice’s Communication, Partnership and Teamwork domain and the Maintaining Trust domain), so that she is made aware of her change in behaviour and its potential impact on others.

How well did you know this?
1
Not at all
2
3
4
5
Perfectly
13
Q
  1. Part A (for reference): You are working on a general medical ward. Over the past week, you have noticed that your FY2 colleague, Eliza, has been more withdrawn than normal. She has also been more easily irritated, which is unlike her typically friendly manner. Earlier today, you noticed that Eliza was very abrupt with one of the student nurses, who looked upset in response. As you enter the staff room to have your lunch, you overhear some of the other nurses complaining about how rude Eliza has been recently. The nurses see you and continue talking.
    Part B: This afternoon you are sat with your FY2 colleague, Eliza, on the ward. She becomes frustrated by making a mistake on some paperwork and disturbs you from writing your own notes. She apologises and explains that she is feeling very stressed at the moment, as she is worrying about an upcoming exam. She tells you that she that has been finding it difficult to concentrate at work and apologises if her recent mood has affected you in any way.
    Rate the appropriateness of the following actions in response to this situation. (1= Very appropriate; 4= Inappropriate).
    A. Reassure your FY2 colleague that she will feel less stressed when the exam is over
    B. Ask your FY2 colleague if she has considered speaking to her educational supervisor* about how she is feeling
    C. Suggest taking on some of your FY2 colleague’s workload to allow her more time to study
    D. Inform your FY2 colleague that some of the nurses have noticed a change in her behaviour
A

A) Reassure your FY2 colleague that she will feel less stressed when the exam is over Correct Key: 2 - Somewhat appropriate
Rationale: This action is somewhat appropriate because although is it is important that you support colleagues who may have problems with their performance or health, reassuring her is beneficial to her in terms of making her feel better. However, this action does not address the impact that her behaviour is having on others and it is also important that you are honest when giving feedback to your colleague about your observations/concerns (General Medical Council [GMC] good medical practice’s Communication, Partnership and Teamwork domain and the Maintaining Trust domain).
B) Ask your FY2 colleague if she has considered speaking to her educational supervisor* about how she is feeling Correct Key: 1 - Very appropriate
Rationale: This action is very appropriate as it is important that you support colleagues who may have problems with their performance or health and suggesting that Eliza discusses how she is feeling is an appropriate suggestion in terms of getting Eliza the support she needs to resolve the challenges that she is facing (GMC good medical practice’s Communication, Partnership and Teamwork domain).
C) Suggest taking on some of your FY2 colleague’s workload to allow her more time to study
Correct Key: 4 - Inappropriate
Rationale: This action is inappropriate because it is out of your scope of practice to take on your colleague’s workload, raising potential risk to patient safety (GMC good medical practice’s Knowledge, Skills and Performance domain and the Patient Safety and Quality domain). The monitoring and supervision of Eliza’s caseload and performance is the responsibility of her clinical/educational supervisor and not you as her peer.
D) Inform your FY2 colleague that some of the nurses have noticed a change in her behaviour
Correct Key: 2 - Somewhat appropriate
Rationale: This action is somewhat appropriate because it is your responsibility to raise concern about your colleague’s behaviour and be honest about its impact on the rest of the team (GMC good medical practice’s Communication, Partnership and Teamwork domain and the Maintaining Trust domain). However, it is also important to consider the potential impact of disclosing this information to Eliza who is already stressed about her workload and upcoming exams.

How well did you know this?
1
Not at all
2
3
4
5
Perfectly
14
Q
  1. You are on a medical team that is well staffed, so your consultant has asked you to help out another understaffed medical team for an afternoon. You have completed the jobs for your patients and would be happy to help another team if needed. However, the ward nurse in charge is very unhappy about losing a team member for the afternoon and asks you to stay on the ward but does not give you a reason.
    Rate the importance of the following considerations in the management of this situation. (1= Very important; 4= Not at all important).
    A. That the ward nurse in charge must have a reason for wanting you stay
    B. That the consultant probably knows the requirements of both wards the best
    C. That you might have an easier afternoon if you stay on your current ward
    D. That your relationship with the ward nurse in charge might be negatively affected if you go to the other ward
A

A) That the ward nurse in charge must have a reason for wanting you stay Correct Key: 2 - Important
Rationale: This is an important consideration because the ward nurse in charge may have a different but equally relevant perspective about clinical priorities which needs to be taken into consideration when reviewing staffing (General Medical Council [GMC] good medical practice’s Communication, Partnership and Teamwork domain). However you have completed all of the jobs for your patients and are able to support the understaffed ward, as requested by your consultant.
B) That the consultant probably knows the requirements of both wards the best Correct key: 1 - Very important
Rationale: This is a very important consideration because your consultant is responsible for providing clinical supervision, support and monitoring of your allocated caseload. Therefore, the consultant probably has a better overview of your capabilities as well as the requirements of both wards (GMC good medical practice’s Communication, Partnership and Teamwork domain) and is best placed to make the decision about where you would be best placed in terms of contribution to the wider multi-disciplinary team.
C) That you might have an easier afternoon if you stay on your current ward Correct Key: 4 - Not at all important
Rationale: This consideration is not at all important because as a doctor, it is your duty to work collaboratively with others to ensure that safe and effective care is delivered to all patients (GMC good medical practice’s Communication, Partnership and Teamwork domain).
D) That your relationship with the ward nurse in charge might be negatively affected if you go to the other ward
Correct Key: 3 - Of minor importance
Rationale: This consideration is of minor importance as patient care and safety must be prioritised above all other factors in the clinical setting (GMC good medical practice’s Patient Safety and Quality domain), including your relationship with the ward’s nurse in charge.

How well did you know this?
1
Not at all
2
3
4
5
Perfectly
15
Q
  1. A patient on the Acute Admissions Unit* (AAU) was admitted to hospital with pancreatitis two weeks ago. The patient was on holiday near the hospital when he was admitted and has therefore asked to be transferred to a hospital closer to his home, which is a three hour drive away. The consultant explains that it is not safe for the patient to travel for at least another week. On your way to the ward, you walk past the patient’s daughter. You notice that she is upset, so you ask her if she is OK. She tells you, “I can’t keep travelling three hours every day to visit my father, I’m too exhausted, but he hasn’t got anyone else. I wish he could be transferred sooner.”
    Rate the appropriateness of the following responses to say in this situation. Assume they are all said in a polite way. (1= Very appropriate; 4= Inappropriate).
    A. “Don’t worry, your father will be transferred next week.”
    B. “Your father’s health is the most important thing to consider in deciding when he can be transferred.”
    C. “It is not necessary for you to visit every day, I am sure he would be OK.”
    D. “Could you speak with your father on the phone, rather than visiting so often?”
A

A) “Don’t worry, your father will be transferred next week.” Correct Key: 4 - Inappropriate
Rationale: This response is inappropriate because although it seems to be providing reassurance, the information is potentially incorrect or inaccurate because there is no guarantee that the patient would be medically stable enough to be transferred in a week’s time (General Medical Council [GMC] good medical practice’s Communication, Partnership and Teamwork domain and the Maintaining Trust domain). The other potential issue is that you are discussing a patient’s confidential clinical information in a public space with his daughter without ascertaining whether the patient is happy for you to do so. Therefore, potentially breaching the patient’s right to confidentiality.
B) “Your father’s health is the most important thing to consider in deciding when he can be transferred.” Correct Key: 1 - Very appropriate
Rationale: This response is very appropriate because it prioritises the patent’s safety and wellbeing as well as his needs over other considerations (GMC good medical practice’s Knowledge, Skills and Performance domain, the Patient Safety and Quality domain and the Communication, Partnership and Teamwork domain).
C) “It is not necessary for you to visit every day, I am sure he would be OK.”
Correct Key: 3 - Somewhat inappropriate
Rationale: This response is somewhat inappropriate because although it comes across as being reassuring, it fails to consider the patient’s specific needs or the needs of his family (GMC good medical practice’s Communication, Partnership and Teamwork domain). It is also based on an assumption that the patient will be okay without necessarily having all the facts required to make such an assertion and without being able to predict the impact on the patient and their family (GMC good medical practice’s Knowledge, Skills and Performance domain and the Patient Safety and Quality domain).
D) “Could you speak with your father on the phone, rather than visiting so often?”
Correct Key: 2 - Somewhat appropriate
Rationale:
This response is somewhat appropriate because it is supporting the patient and his family with information, however the daughter needs to make an informed decision about his ongoing healthcare needs and how to support him (GMC good medical practice’s Communication, Partnership and Teamwork domain).

How well did you know this?
1
Not at all
2
3
4
5
Perfectly
16
Q
  1. You are being shadowed by a medical student, Jeremy, whilst consulting an elderly patient, Mrs McDermott. You are explaining to her that she will be prescribed several medications, when Jeremy interrupts to say that he thinks there is an alternative drug, which would be more appropriate for Mrs McDermott than the one you have suggested. He says that he has read about this in an article online. Mrs McDermott looks confused.
    Rate the appropriateness of the following actions in response to this situation. (1= Very appropriate; 4= Inappropriate).
    A. Apologise to Mrs McDermott for causing confusion following Jeremy’s comment
    B. Ignore Jeremy’s comment and continue the conversation with Mrs McDermott
    C. After the consultation, explain to Jeremy that it was an inappropriate time to bring up the article that he has read online
    D. Ask Jeremy to explain to the patient why he believes that the alternative drug he has suggested is more effective
A

A) Apologise to Mrs McDermott for causing confusion following Jeremy’s comment Correct Key: 1 - Very appropriate
Rationale: This action is very appropriate as it is important to be honest, acknowledge when things go wrong in clinical practice and to apologise in the best interest of maintaining trust in you as a doctor and in the profession (General Medical Council [GMC] good medical practice’s Communication, Partnership and Teamwork domain and the Maintaining Trust domain).
B) Ignore Jeremy’s comment and continue the conversation with Mrs McDermott Correct Key: 4 - Inappropriate
Rationale: This action would be inappropriate as it fails to address the confusion that Jeremy’s interruption has caused which may negatively impact on your ability to establish a meaningful working relationship with Mrs McDermott (GMC good medical practice’s Communication, Partnership and Teamwork domain and the Maintaining Trust domain).
C) After the consultation, explain to Jeremy that it was an inappropriate time to bring up the article that he has read online
Correct Key: 1 - Very appropriate
Rationale: This action is very appropriate because it is your responsibility as a doctor to support students and other colleagues in practice to develop their clinical and professional skills. Providing Jeremy with honest feedback on the impact of his interruption on the patient should encourage him to be more aware of the impact of his actions on others in the future (GMC good medical practice’s Communication, Partnership and Teamwork domain and the Maintaining Trust domain).
D) Ask Jeremy to explain to the patient why he believes that the alternative drug he has suggested is more effective
Correct Key: 4 - Inappropriate
Rationale: This action is inappropriate as you may be asking Jeremey to carry out a task that is outside his scope of practice as a student (GMC good medical practice’s Knowledge, Skills and Performance domain), potentially exposing the patient to risk (GMC good medical practice’s Patient Safety and Quality) caused by the confusion from the contrasting information from an unverified source being presented to her. This interaction may have a detrimental effect on the patient’s trust in you as her doctor as well as the wider profession/team, resulting in reduced ability to engage or establish a successful working relationship with the clinical team (GMC good medical practice’s Maintaining Trust domain).

How well did you know this?
1
Not at all
2
3
4
5
Perfectly
17
Q
  1. Part A: You are covering a ward out of hours. An FY2 doctor, Raphael tells you that he will be back shortly. As the ward becomes busier, you note that Raphael has not returned from his break. You find Raphael asleep in the break room. You are very busy and becoming overwhelmed on the ward, so you decide to wake Raphael. Raphael tells you to leave him alone as he needs the rest.
    Rate the appropriateness of the following actions in response to this situation. (1= Very appropriate; 4= Inappropriate). A. Explain to Raphael that you feel overwhelmed, under pressure and need his support
    B. Explain to Raphael that his behaviour is inappropriate
    C. Leave Raphael to sleep and carry on alone throughout the shift
    D. Ask other colleagues on the ward for their support
    E. Inform a more senior colleague that you are not receiving the support that you need from your FY2
    F. Decide to work late to finish the tasks
A

A) Explain to Raphael that you feel overwhelmed, under pressure and need his support Correct key: 1 - Very appropriate Rationale: This is a very appropriate action. It is clear and explicit that you need help on the ward, as you cannot manage the workload alone, whilst remaining polite and professional. It does not cast judgement on why Raphael is sleeping, but simply asks for his help now that he is awake. B) Explain to Raphael that his behaviour is inappropriate Correct key: 2 - Somewhat appropriate Rationale: This action is somewhat appropriate. Although staff members are entitled to take breaks and rest, which may even include a small nap, this must not over run to when they are meant to be performing clinical duties. However, the response is not entirely appropriate as although Raphael’s behaviour is inappropriate, it does not explain to him what your immediate concerns are with the busy ward and that his help is needed now. C) Leave Raphael to sleep and carry on alone throughout the shift Correct key: 4 - Inappropriate Rationale: This is an inappropriate action. Regardless of Raphael’s reasons for taking a nap, when his allotted break time has finished, Raphael should be ready to continue his clinical duties. If he is unwell or has genuine reasons, then the situation can be escalated, so that he can be sent home and appropriate cover can be found. However, by leaving him to sleep, the situation is not being dealt with, and it also means that the clinical work is becoming overwhelming for one FY1 doctor. Additionally, patient safety may be at risk due to not enough staff and support available. D) Ask other colleagues on the ward for their support Correct key: 2 - Somewhat appropriate Rationale: This action is somewhat appropriate. It is recognising your own limitations and that you are becoming overwhelmed and require support on the ward. Asking for help is a positive step and ensures patient safety. However, the colleagues on the ward may have other duties and asking for their help is taking them away from their own work and it hasn’t dealt with the main problem at this time, which is Raphael sleeping and not able to perform his own duties.
E) Inform a more senior colleague that you are not receiving the support that you need from your FY2 Correct key: 1 - Very appropriate Rationale: This is a very appropriate action. Escalating the situation appropriately has two benefits: it means that additional support can be found for you in order to carry out the jobs properly, and patient safety isn’t compromised. It also means that a senior colleague can deal sensitively with Raphael, as there may be more complex issues involved that may explain the reasons why he is so tired and unable to complete his shift, which can be dealt with by his senior. F) Decide to work late to finish the tasks
Correct key: 4 - Inappropriate Rationale: This is an inappropriate action. Despite the good intentions behind working late to get the jobs completed, it is inappropriate because, as an individual, you cannot simply take on an entire colleague’s workload. You will be tired after a long shift, and in these circumstances mistakes can happen. Rest periods are essential, so you need to finish your shift when allocated and get the appropriate rest needed, so that you are ready for your next shift. Safe working times and rotas have been designed with this in mind. Working late to finish tasks also sets a precedent for other doctors to do the same. Any leftover jobs can be safely handed over to the on-call team or postponed until the following day. Finally, by working late to finish the tasks, not only might it be unsafe, but it is not dealing with the problem that you have been unsupported by a team member, Raphael, who is asleep, and these needs escalating appropriately.

How well did you know this?
1
Not at all
2
3
4
5
Perfectly
18
Q
  1. Part A (for reference) You are covering a ward out of hours. An FY2 doctor, Raphael tells you that he will be back shortly. As the ward becomes busier, you note that Raphael has not returned from his break. You find Raphael asleep in the break room. You are very busy and becoming overwhelmed on the ward, so you decide to wake Raphael. Raphael tells you to leave him alone as he needs the rest.
    Part B: Raphael remains asleep for the remainder of the shift. As you are leaving, Raphael wakes up and tells you that, before that shift, he had not slept in 4 days. He commends you for managing well without him. Later on, you find out that your colleague, Shannon, has also had a previous similar experience with Raphael.
    Rate the appropriateness of the following actions in response to this situation. (1= Very appropriate; 4= Inappropriate). A. Ask Raphael why he is struggling to sleep during his time off B. Tell Raphael that he is putting patients at risk by coming to work over-tired C. Encourage Raphael to seek support from his General Practitioner (GP)* D. Advise Raphael that you think it is inappropriate to sleep whilst on shift
A

A) Ask Raphael why he is struggling to sleep during his time off Correct key: 1 - Very appropriate Rationale: This is a compassionate approach, understanding that there are likely to be reasons why Raphael has been sleeping on his shifts. As an individual, you may not be able to fix the reasons on your own, and you will still need to escalate the situation, but by showing an empathetic and compassionate response to a fellow colleague, you are more likely to get a positive response from Raphael, and foster trust and positive working relationship in future. B) Tell Raphael that he is putting patients at risk by coming to work over-tired
Correct key: 1 - Very appropriate Rationale: This is a very appropriate action. Whatever the reasons are for Raphael being so tired that he needs to sleep through his shift, in doing so, he is risking patient safety. If he has genuine reasons, these need to be escalated and dealt with appropriately and he may require a period of sick or compassionate leave before he is safe to work again. C) Encourage Raphael to seek support from his General Practitioner (GP)*
Correct key: 1 - Very appropriate Rationale: This is a kind and compassionate approach, taking into consideration that there may be genuine reasons why Raphael is sleeping during a shift and these reasons need addressing, as he is currently not safe to be at work. Encouraging Raphael to seek support from his GP is a compassionate and empathetic approach by directing him to get some help, rather than cast judgments on his behaviour. D) Advise Raphael that you think it is inappropriate to sleep whilst on shift
Correct key: 2 - Somewhat appropriate Rationale: Although it is inappropriate for Raphael to sleep whilst on shift (not in his allocated break time), advising him that you think his behaviour in inappropriate is somewhat unhelpful, as it might be interpreted as inflammatory by Jeremy and cause a breakdown in trust. This action does not offer any practical solutions, such as speaking with a senior colleague or healthcare professional, nor does it highlight to Raphael the additional stress that it has put you under, which has meant that jobs on the ward are not completed and thus patient safety may be compromised.

How well did you know this?
1
Not at all
2
3
4
5
Perfectly
19
Q
  1. Whilst working on a surgery rotation, your consultant asks you to prescribe a high dose of warfarin for a patient to go home with. However, the pharmacist informs you that the hospital protocol advises to use a smaller dose of warfarin to begin with, to reduce the risk of serious haemorrhage. You have only recently started working with your consultant, but your FY1 colleagues have told you that she often gets angry when her decisions are challenged.
    Choose the THREE most appropriate actions to take in this situation. * Option A: Ask the pharmacist to supply the high dose of warfarin, as per the prescription * Option B: Inform your consultant that the warfarin prescription is against the hospital protocol * Option C: Ask your consultant if she wants you to change the prescription to a smaller dose * Option D: Ask the patient whether he is willing to accept the risks associated with the higher dose * Option E: Change the dose of warfarin to follow the hospital protocol without speaking to the consultant * Option F: Ask the pharmacist to contact your consultant to discuss the dosage directly with her * Option G: Ask a different senior colleague for advice on how to proceed with the prescription * Option H: Ask your consultant to explain why she asked you to prescribe a high dose of warfarin
A

Correct Key: CFH
Rationale: The General Medical Council (GMC) standards of practice require all medics to prioritise safety. In practice, this is done by adhering to evidence practice and following guidelines and protocols, therefore, Option C: Ask your consultant if she wants you to change to a smaller dose as per the Trust protocol would be an appropriate action to take in this situation.
In view of the consultant’s reputation for reacting angrily when challenged, it may help matters to get the pharmacist as the expert on medication to challenge her decision based on the Trust protocol, therefore, Option F: Ask the pharmacist to contact your consultant to discuss the dosage directly with her would be another appropriate action to take. According to the GMC, Communication is a key requirement of good medical practice: The other course of action is to discuss the matter with the consultant to understand as a trainee, the rationale for her decision and the atypical prescription. This may serve to be a useful/meaningful learning opportunity as a trainee, therefore, Option H: Ask your consultant to explain why she asked you to prescribe a high dose of warfarin would be another appropriate action to take in this situation.

How well did you know this?
1
Not at all
2
3
4
5
Perfectly
20
Q
  1. You are working in the Emergency Department. A female patient approaches you as she is leaving and tells you that she thought you were very rude during the consultation that you had with her earlier. The patient is angry because she noticed that you were looking at your phone during the consultation, however, you were looking at her observations on your hospital phone.
    Choose the THREE most appropriate actions to take in this situation. * Option A: Reassure the patient that you were paying attention during her consultation * Option B: Apologise to the patient for appearing rude during her consultation * Option C: Explain to the patient that you were looking at her observations on your hospital phone * Option D: Tell the patient that you were using your hospital phone so that she could receive the best possible care * Option E: Advise the patient that in future you will tell patients why you are checking your phone during consultations * Option F: Explain to the patient that doctors frequently have to check information on their hospital phones * Option G: Show the patient her observations that you were looking at on your hospital phone * Option H: Ask the patient if she would like to raise the use of hospital phones with one of your senior colleagues
A

Correct Key: BCE
Rationale: The General Medical Council’s (GMC’s) standards of practice includes communication and partnership working as being integral components of good medical practice. Specifically, working in partnership with patients by being able to listen to and respond to their concerns and preferences (GMC good medical practice’s Communication, Partnership and Teamwork domain). Therefore, Option B: Apologise to the patient for appearing rude during her consultation, would be an appropriate action to take in this situation. An explanation of what was happening would be appropriate in terms of reassuring the patient about what was happening during their care, therefore, Option C: Explain to the patient that you were looking at her observations on your hospital phone would be another appropriate action to take. In the best interest of the working relationship with the patient, it is appropriate to respond to feedback in terms of informing future practice through lessons learnt from the feedback, therefore another appropriate action in this situation would be Option E: Advise the patient that in future you will tell patients why you are checking your phone during consultations. This course of action may also serve to reassure and rebuild trust between the patient and clinician.

How well did you know this?
1
Not at all
2
3
4
5
Perfectly
21
Q
  1. You are an FY1 doctor working in the general surgical department of a small hospital. The registrar/specialty trainee* contacts you and asks you to inform a patient, Mr Jones, that his operation will be cancelled due to an emergency case. The registrar/specialty trainee tells you he is in theatre and cannot speak to Mr Jones himself. Mr Jones is on the ward, accompanied by his relatives. When you inform him of the situation, Mr Jones looks upset and his relatives gets very angry. They are unhappy with the situation because Mr Jones’ operation has already been cancelled twice. The relatives begin shouting about the poor care delivered by your hospital and demand an explanation.
    Choose the THREE most appropriate responses to say in this situation. Assume they are all said in a polite way. * Option A: “I’m sorry but there is an emergency case that takes priority.” * Option B: “I can see you are upset. I know this must be disappointing.” * Option C: “I agree, this is not the best outcome.” * Option D: “Is there anything else I can do to help?” * Option E: “The registrar/specialty trainee has made the decision; I can not do anything about it I’m afraid.” * Option F: “You can wait to discuss this further with the registrar/specialty trainee if you would prefer?” * Option G: “I understand your frustration but getting angry will not help.” * Option H: “There is no need to shout at me, I can not change anything.”
A

Correct Key: BDF
Rationale: The General Medical Council’s (GMC) standards of practice include the ability to listen to and respond to patients concerns and preferences (GMC good medical practice’s Communication, Partnership and Teamwork domain). Therefore, an appropriate response to say in this situation would be Option B: “I can see you are upset. I know this must be disappointing.” as it acknowledges the effect of the cancellation of the operation on the patient and Mr Jones’s family, validating their collective experience of the care Mr Jones had received so far. Another appropriate response would be, Option D: “Is there anything else I can do to help whilst you wait for your operation to be rearranged?”, as it acknowledges the reality of the operation being cancelled as being unavoidable while reassuring the patient and his family that the operation is to be rescheduled and you are available to provide support (GMC good medical practice’s Communication, Partnership and Teamwork domain). In some cases, as the messenger, the only way to resolve the situation is to defer to the primary source of the information, particularly, if they are a more senior colleague and the primary decision maker, therefore, another appropriate response in this situation is, Option F: “You can wait to discuss this further with the registrar/specialty trainee if you would prefer?”, which offers Mr Jones and his family, the option of speaking with the more senior colleague/decision maker if further explanation/clarification is required (GMC good medical practice’s Communication, Partnership and Teamwork domain).

How well did you know this?
1
Not at all
2
3
4
5
Perfectly
22
Q
  1. You are in the office with your FY2 colleague who is collecting feedback, via email, from his colleagues to use for his learning portfolio. He sends a request to another staff member who is also his girlfriend, who you have previously worked with. He tells you that this is an easy way to make sure that he gets good feedback.
    Choose the THREE most appropriate actions to take in this situation. * Option A: Suggest to your FY2 colleague that he requests feedback from someone else instead of his girlfriend * Option B: Contact his girlfriend, recommending that she does not complete the feedback form * Option C: Ask your FY2 colleague if he thinks his girlfriend will respond to his request for feedback * Option D: Report this incident to your FY2 colleague’s educational supervisor
    * Option E: Inform your FY2 colleague that his behaviour is unprofessional * Option F: Explain the benefits of receiving constructive feedback to your colleague * Option G: Arrange to discuss the situation with your educational supervisor * Option H: Offer to provide feedback to your FY2 colleague yourself
A

Correct Key: AFG
Rationale: The General Medical Council (GMC) states that being honest, open and acting with integrity in practice are essential characteristics of good medical practice (GMC good medical practice’s Communication, Partnership and Teamwork and Maintaining Trust domains). Therefore, Option A: Suggest to your FY2 colleague that he requests feedback from someone else instead of his girlfriend, would be an appropriate action to take in this situation as this action may encourage your FY2 colleague to remember the importance of demonstrating those values in his actions. As a trainee and a clinician, feedback is a valuable and compulsory part of both personal and professional development, as feedback identifies areas of strength and areas of deficit, requiring further support or development. Therefore, Option F: Explain the benefits of receiving constructive feedback to your colleague, would be another appropriate action to take in this situation (GMC good medical practice’s Communication, Partnership and Teamwork domain). Raising concern about poor practice is also a professional responsibility and, in this case, it is your responsibility and duty to bring your FY2 colleague’s behaviour to the attention of a more senior colleague (GMC good medical practice’s Safety and Quality domain). Therefore, another appropriate course of action to take in this situation would be, Option G: Arrange to discuss the situation with your educational supervisor.

How well did you know this?
1
Not at all
2
3
4
5
Perfectly
23
Q
  1. You are working on an orthopaedic ward. Your fellow FY1 colleague does not like going to the operating theatre and has been avoiding it. As a result, you have spent more time assisting in theatre, and have missed some bedside teaching sessions arranged by the registrar/specialty trainee*. The registrar/specialty trainee has noticed and has made a remark about the time you are away from the ward. You are now meeting with your FY1 colleague to discuss your allocation
    Choose the THREE most appropriate responses to say in this situation. Assume they are all said in a polite way.
    * Option A: “The registrar is not happy that I have missed some bedside teaching sessions.”
    * Option B: “I’ve missed all the bedside teaching because I’m often in theatre.”
    * Option C: “I would like us to split the work more evenly in future so that we both can meet all of our learning objectives.”
    * Option D: “Is there anything I can do to help, to make you feel better about going to theatre?”
    * Option E: “I think you should talk to your supervisor about why you have been avoiding theatre.” * Option F: “You choosing to spend more time on the ward is having an impact on both of us.” * Option G: “I understand you do not like going to theatre, but it is an important part of the job.” * Option H: “The registrar will be disappointed if he notices that I’ve only been in theatre because you did not want to go.”
A

Correct Answer Key: CDF
Rationale: Option C is an appropriate response as it lets you clearly say that there should be an equal distribution of work going forward. This would work irrespective of any personal equations you may share with your colleague and remains very professional. Option D would be another appropriate response as it offers help and seeks to clarify any issues the colleague may be having. Option F would also be an appropriate response to say as it states what the problem is and would be a good way to check if your colleague has any insight into the issues they may inadvertently be causing.
Options A and B do not offer sufficient clarity to your colleague that he may be causing an issue and make it seem more like a personal problem. Options E, G and H would appear to be more confrontational and make your colleague defensive rather than help solve the issue.

How well did you know this?
1
Not at all
2
3
4
5
Perfectly
24
Q
  1. Part A You are conducting a ward round with your FY2 colleague, Dr Lee. While reviewing one of your patients, Patricia, you see that her chart indicates that she has refused medication for the past week. It is not critical for her short-term health, however, her symptoms are unlikely to improve if she does not take her medication. Dr Lee says, “You don’t seem to care about getting better, you may as well go home” and leaves the room. Patricia looks upset by Dr Lee’s comment.
    Choose the THREE most appropriate responses to say in this situation. Assume they are all said in a polite way. * Option A: “Is there a reason why have you been refusing to take your medication?” * Option B: “I really think that you should listen to the advice that Dr Lee is giving you.” * Option C: “Do you not want to get any better?” * Option D: “Do not worry, there are no short term impacts of you not taking your medication.” * Option E: “Are there any questions that you would like to ask me about your medication?” * Option F: “I’m sorry, Dr Lee should not have spoken to you like that.” * Option G: “Do not get upset, if you want to talk to me, I am here to help.” * Option H: “It is really important that you take your medication if you want to get better.”
A

Correct Key: AEF
Rationale: As part of good medical practice, the General Medical Council (GMC) highlight the importance of working in partnership with patients, including, listening to and responding to their concerns and preferences (GMC good medical practice’s Communication, Partnership and Teamwork domain). Therefore, Option A: “Is there a reason why have you been refusing to take your medication?”, would be an appropriate response to say in this situation as it may enable you to understand the rationale for Patricia’s choice in declining her medication. In most cases of non-compliance with treatment, research has shown that fear or health related anxiety may be a significant contributory factor to patients not following medical advice or instruction. Addressing what those concerns may be a significant step towards allaying those fears/anxieties and perhaps facilitating cooperation and compliance with care. Therefore, Option E: “Are there any questions that you would like to ask me about your medication?”, would be another appropriate response to say in this situation. Good medical practice also requires being honest, open and acting with integrity as per the duty of candour. Therefore, Option F: “I’m sorry, Dr Lee should not have spoken to you like that.”, acknowledges that something has gone wrong in the course of Patricia’s care and apologises for it. This in turn serves to maintain the patient’s trust in you and the profession (GMC good medical practice’s Maintaining Trust domain).

How well did you know this?
1
Not at all
2
3
4
5
Perfectly
25
Q
  1. Part A (for reference): You are conducting a ward round with an FY2 Doctor, Dr Lee. While reviewing one of your patients, Patricia, you see that Patricia’s chart indicates she has refused medication for the past week. It is not critical for her short-term health, however, her symptoms are unlikely to improve if she does not take her medication. Dr Lee states, “what a waste of space it is you being here Patricia. You may as well go home if you do not want to get better” and leaves the room. Patricia looks upset by Dr Lee’s comment. Part B: Dr Lee approaches you after you have spoken with Patricia and expresses his regret for the way he spoke to Patricia. He explains that he is under pressure from the consultant to discharge patients as quickly as possible. He tells you that he feels that this has had a negative impact on his relationships with many of his patients. You have not yet finished the ward round with Dr Lee.
    Choose the THREE most appropriate actions to take in this situation. * Option A: Suggest to Dr Lee that he takes a break before finishing the ward round * Option B: Speak to your clinical supervisor* about Dr Lee’s behaviour towards Patricia * Option C: Ask your colleagues on the ward to monitor Dr Lee’s behaviour with patients * Option D: Explain to Dr Lee that you understand that he is under a lot of pressure * Option E: Suggest that Dr Lee speak to the consultant about his concerns * Option F: Advise Dr Lee apologises to Patricia for the way he spoke to her * Option G: Suggest to Dr Lee that he stops trying to discharge patients early and focuses on maintaining positive relationships * Option H: Ask Dr Lee if there is anything that you can do to support him
A

Correct Key: EFH Rationale: According to the General Medical Council (GMC), a key aspect of good medical practice is ensuring that doctors work within their limitations and are prompt in raising concerns about practice (GMC good medical practice’s Safety and Quality domain). Therefore, Option E: Suggest that Dr Lee speak to the consultant about his concerns, is an appropriate action to take in this situation.
In the interest of his working relationship with his patients, an important step for Dr Lee to take would be to acknowledge and apologise for his actions, which may serve to re-establish Patricia’s trust in him and enable them to have a positive and effective patient-doctor partnership (GMC good medical practice’s Communication, Partnership and Teamwork and Maintaining Trust domains). Therefore, Option F: Advise Dr Lee to apologise to Patricia for the way he spoke to her, would be another appropriate action to take in this situation. The GMC emphasises the role of teamwork as an important aspect of good medical practice (GMC good medical practice’s Communication, Partnership and Teamwork domain). An integral part of teamwork is supporting each other and working together to achieve a common aim/goal. Therefore, Option H: Ask Dr Lee if there is anything that you can do to support him, would be another appropriate action to take in this situation. This action may serve to reassure Dr Lee that he is not alone and has the support of his colleague(s).

How well did you know this?
1
Not at all
2
3
4
5
Perfectly
26
Q
  1. You are clerking* in a three year old patient with cystic fibrosis* who attends frequently with pulmonary* complications. You discover that her parents smoke cigarettes heavily at home and you mention to them that this may be a contributing factor to their daughter’s deterioration. Later that day, the parents tell you they intend to make an official complaint about you because they feel like you were blaming them for their daughter’s deterioration.
    Choose the THREE most appropriate responses to say in this situation. Assume they are all said in a polite way. * Option A: “I am sorry that I offended you with my comment, but making a complaint will not help your daughter.” * Option B: “Have you considered the benefits of stopping smoking?” * Option C: “I did not mean to offend you with my comment earlier.” * Option D: “I understand this situation with your daughter must be difficult for your family.” * Option E: “Do you have any other suggestions about what is causing your daughter to have frequent pulmonary complications?” * Option F: “Please give me the opportunity to explain the facts about smoking and pulmonary complications.” * Option G: “Was there something specific that I said that upset you?” * Option H: “My intentions are to inform you of how you can help your daughter.”
A

Correct Key: CDF
Rationale:
A key component of the General Medical Council (GMC) good medical practice’s Communication, Partnership and Teamwork domain includes treating patients politely and considerately. In its Working in Partnership with Patient’s domain, the GMC also highlights the importance of listening to and responding to patients’ concerns and preferences. In view of these two domains, an appropriate response to say in this situation is, Option C: “I did not mean to offend you with my comment earlier”, which addresses both requirements. Acknowledging the parents’ perspective is essential to fostering a positive working relationship or partnership, therefore, Option D: “I understand this situation with your daughter must be difficult for your family”, would be another appropriate response to say in this situation as this is polite, while demonstrating consideration and compassion. The GMC good medical practice’s Working in Partnership with Patient’s domain also outlines the importance of providing patients with information that they want or need in a way they can understand. Therefore, Option F: “Please give me the opportunity to explain the facts about smoking and pulmonary complications”, would be another appropriate response to say in this situation.

How well did you know this?
1
Not at all
2
3
4
5
Perfectly
27
Q
  1. You are about to start a night shift on a medical ward. During handover, Hussein, the FY1 doctor who has been working the day shift, tells you that he has nothing to hand over. When you are on the ward later that evening, a senior nurse informs you that she bleeped* Hussein several times during the day and spoke to him over the telephone about an 88 year old patient with severe chest pain. You review and treat the patient accordingly. When you discuss this incident with Hussein the next day, he states that he was never informed about this patient, and that the nurse is lying.
    Choose the THREE most appropriate actions to take in this situation.
    * Option A: Inform Hussein’s clinical supervisor* of the allegations that he has made about * the senior nurse * Option B: Explain the situation to your clinical supervisor, seeking advice on how to * proceed * Option C: Advise Hussein of the severity of his allegations, ensuring that he is aware of * what he is saying * Option D: Inform the senior nurse that Hussein told you that she was lying about the patient * with chest pain * Option E: Speak to the senior nurse involved, obtaining further details about the incident * Option F: Explore with the other nurses whether they overheard the telephone call between * Hussein and the senior nurse * Option G: Suggest to Hussein that he talks to the senior nurse about the incident * Option H: Fill in a critical incident form*, documenting the incident
A

Correct Key: BCG
Rationale: This question tests your professionalism in the context where two colleagues (the senior nurse and Hussein) have each made allegations about each other professionalism. It is appropriate to emphasise the severity of the allegations when talking to Hussein (option C), and seek early senior advice (option B) on how to proceed in this situation. (Option G) may be uncomfortable for Hussein to consider given the allegations, however, suggesting a face-to-face discussion with the nurse may help resolve this situation particularly if a simple misunderstanding (e.g. a malfunctioning pager) has led each party to make allegations about the other.
It is not your place as the FY1 to initiate an investigation (options E, F and H) into the incidents which have occurred and similarly not your place to raise allegations on Hussain’s behalf (option A). Accusing the senior nurse of lying (option D) is likely to be counterproductive, unprofessional and embroil you in the conflict.

How well did you know this?
1
Not at all
2
3
4
5
Perfectly
28
Q
  1. Part A While working in a busy surgical ward, your FY1 colleague, Abigail, approaches you in the staff room during your break. She tells you that she is feeling very stressed because of the current workload. Abigail explains that she feels that the stress is negatively affecting her mental health.
    Choose the THREE most appropriate actions to take in this situation.
    * Option A: Ask Abigail if she wants to arrange to talk more about how she feels outside of work
    * Option B: Suggest to Abigail that she takes some time off work until she feels less stressed
    * Option C: Ask Abigail if she has thought about seeing her GP*
    * Option D: Inform Abigail that it is normal to feel stressed when working on a busy ward
    * Option E: Suggest to Abigail that she should ask to reduce her workload if it is affecting her mental health
    * Option F: Advise Abigail to speak to her educational supervisor* about how she is feeling
    * Option G: Suggest to Abigail that she monitors how she is feeling over the next couple of weeks
    * Option H: Ask Abigail if there is anything you can do to help her to feel less stressed
A

Correct Key: CFH
Rationale: As a colleague it is your professional responsibility to provide peer support for a colleague as well as promptly flag up any concerns that may impact on patient care and safety (General Medical Council [GMC] good medical practice’s Communication, Partnership and Teamwork and Safety and Quality domains). Therefore, Option C: Ask Abigail if she has thought about seeing her GP, would be an appropriate action to take in this situation as it is the best way to ensure that Abigail has access to the healthcare she needs for her mental health.
All doctors have a duty to ensure that they are fit to practice, including taking responsibility for their own health as well as others’. It is important to flag up any issues that may impact on their ability to deliver safe and effective healthcare to patients. Therefore, another appropriate action to take in this situation would be, Option F: Advise Abigail to speak to her educational supervisor
about how she is feeling, as this will ensure that Abigail is able to access the appropriate support she needs with managing the current workload allocated to her.
Peer support is an important aspect of teamwork and as part of a Multi-Disciplinary Team (MDT), doctors are expected to support each other and other colleagues (GMC good medical practice’s Safety and Quality and the Communication, Partnership and Teamwork domains). Therefore, another appropriate action to take in this situation is Option H: Ask Abigail if there is anything you can do to help her to feel less stressed as this offers much needed support to Abigail and ensures that she knows she is not alone.

How well did you know this?
1
Not at all
2
3
4
5
Perfectly
29
Q
  1. Part A (for reference): While working in a busy surgical ward, your FY1 colleague, Abigail, approaches you in the staff room during your break. She tells you that she is feeling very stressed because of the current workload. Abigail explains that she feels that the stress is negatively affecting her mental health.
    Part B: The next day, you and Abigail have twenty patients on your list. You divide your list into ten patients each to manage the jobs. Two hours before you are both due to finish, Abigail is visibly upset and tells you she cannot carry on as she feels so stressed. She gives you her list and bleep* and tells you that she is going to leave the hospital now. You have not finished the jobs for your patients, and you know that she has outstanding jobs on her list to complete.
    Choose the THREE most appropriate actions to take in this situation.
    * Option A: Ask Abigail if she can help you to prioritise the most important jobs
    * Option B: Tell Abigail that you need her to finish her jobs before she can leave
    * Option C: Advise Abigail that you have your own jobs to finish and cannot complete her jobs if she leaves
    * Option D: Ask Abigail if she is sure that she cannot carry on working
    * Option E: Suggest to Abigail that she takes a ten minute break and then sees if she can continue working
    * Option F: Inform Abigail that she will need to tell other ward staff if she intends to leave now
    * Option G: Ask Abigail how she intends to make sure that her jobs are completed if she leaves now
    * Option H: Ask Abigail if she would feel better if she swapped some jobs with you
A

Correct Key: AEH
Rationale: The General Medical Council (GMC) prioritises patient safety as core to good medical practice and part of that outlines the doctor’s responsibility to ensure that their allocated workload is covered appropriately, including prioritisation of need (GMC good medical practice’s Safety and Quality and the Communication, Partnership and Teamwork domains). Therefore, an appropriate action to take in this situation is, Option A: Ask Abigail if she can help you to prioritise the most important jobs.
The clinical environment can often be challenging and stressful, particularly when a clinician is stressed, fatigued or unwell. Research shows that having appropriate support, including peer support around a person during periods when they are experiencing challenging times or crisis is beneficial and increases the chances of a positive outcome: Part of the responsibilities for a doctor includes peer support, including practical advice and assistance with clinical tasks, particularly when patient and/or staff safety may be at risk (GMC good medical practice’s Communication, Partnership and Teamwork and Safety and Quality domains). Therefore, another appropriate action is, Option E: Suggest to Abigail that she takes a ten minute break and then sees if she can continue working, as this may provide Abigail with the much needed temporary pause to have a break, recuperate/regroup and have the opportunity to think about the best course of action for herself.
Problem solving collaboratively is an integral part of clinical practice and in this situation, Option H: Ask Abigail if she would feel better if she swapped some jobs with you, would be another appropriate action in this situation as it offers both you and Abigail the opportunity to work together to manage the workload safely and in a manner that would be both supportive and beneficial for Abigail and the patients.

How well did you know this?
1
Not at all
2
3
4
5
Perfectly
30
Q
  1. You have just started a night shift on the Medical ward. You are bleeped* by a nurse to attend to an 82 year old patient, Mrs Ratnaike, who is complaining of pain. Mrs Ratnaike is crying and tells you that your FY1 doctor colleague, Haroon, was very rough when he inserted a cannula, just before he hurried away. Mrs Ratnaike says that Haroon mentioned that he was late for a social engagement, and she thinks that he was rushing to leave.
    Choose the THREE most appropriate actions to take in this situation.
    * Option A: Telephone Haroon, explaining that he made Mrs Ratnaike cry
    * Option B: Apologise to Mrs Ratnaike for Haroon’s behaviour
    * Option C: Advise Mrs Ratnaike of the hospital’s complaints procedure
    * Option D: Explore with your FY1 doctor colleagues whether they have ever witnessed
    * similar behaviour from Haroon
    * Option E: Ensure that Mrs Ratnaike is comfortable
    * Option F: Inform your consultant of the incident
    * Option G: The next time he is at work, inform Haroon that he made Mrs Ratnaike cry
    * Option H: Reassure Mrs Ratnaike that Haroon would not rush a medical procedure
A

Correct Key: BEG
Rationale: In this situation, you have heard about the patient’s complaint, but have not heard your colleague’s side of the story. It would be sensible to apologise to the patient, as you are dealing directly with the patient (option B) and to check that she is now comfortable (option E). Following this it would be appropriate to inform your colleague of the event the next time you see him (option G) so that he may reflect on the event. Advising the patient of the hospital’s complaint procedure would also be an acceptable option (C) and could well constitute the next step, however the patient has not said that she wishes to make a complaint, therefore options B, E, and G comprise the most appropriate course of action.
There is no need to call your colleague after he has left the hospital (option A), as there is no urgency for him to know about and resolve the issue if he has already left the hospital. As this is currently a one-off event, gathering more information from your colleagues (option D) or informing your consultant of the incident (option F) are not yet required. You do not know whether Haroon rushed the medical procedure, so it would be unwise to advise her of this at this stage (option H).

How well did you know this?
1
Not at all
2
3
4
5
Perfectly
31
Q
  1. You have been in your new medical rotation for one month and have noticed that one of your FY1 colleague is consistently late for his shift. This has caused delay to the morning handover and the start of the ward round. As a result of this delay, the ward team tend to finish work late on most days.
    Choose the THREE most appropriate actions to take in this situation.
    * Option A: Report your FY1 colleague to his clinical supervisor*
    * Option B: Arrange a private conversation with your FY1 colleague to explain the impact of his lateness
    * Option C: Monitor your FY1 colleague’s behaviour over the coming weeks to see if he continues to be late
    * Option D: Discuss the situation with the other FY1 colleagues on the ward
    * Option E: Inform your colleagues on the ward that you have noticed that one of your FY1 colleague is consistently late
    * Option F: Suggest to the team that you start the morning handover on time, even if someone is missing
    * Option G: Ask your FY1 colleague if he is having any difficulties getting into work on time
    * Option H: Ask your FY1 colleague if he is clear on what time the morning handover and ward round starts
A

Correct Key: BGH
Rationale: Providing feedback and raising concern about conduct is part of the professional responsibilities for all doctors, including providing feedback or raising concern with colleagues (General Medical Council [GMC] good medical practice’s Safety and Quality and the Communication, Partnership and Teamwork domains). Therefore, Option B: Arrange a private conversation with your FY1 colleague to explain the impact of his lateness would be an appropriate action to take in this situation.
Although your FY1 colleague’s repeated actions have had a negative impact on the team and the ward, it is also important to consider that he may have extenuating circumstances such as additional responsibilities, caring responsibilities, childcare obligations or health challenges that may be causing him to be consistently late for work. Therefore, Option G: Ask your FY1 colleague if he is having any difficulties getting into work on time, would be another appropriate action to take in this situation. This may enable your colleague to seek further support if required such as utilising the option to apply for flexible working.
It is also important to consider that minor details such as shift start times for the new medical rotation may not have been conveyed to your colleague as part as his induction to the new rotation. Therefore, Option H: Ask your FY1 colleague if he is clear on what time the morning handover and ward round starts, would be another appropriate course of action in this situation.

32
Q
  1. A 54 year old patient, Kathryn, attends the Emergency Department with an ankle injury. She has attended with a young man, Tristan, who you assume to be her son. You tell Kathryn that hopefully she will be able to go home in the next few days, and ask Tristan whether he will be able to look after his mum when she is at home. Kathryn angrily tells you that Tristan is her husband, not her son. She goes on to say, “It doesn’t matter that Tristan’s age is 22. I don’t know why some people feel the need to be so judgemental. I’d like to speak with someone else please.”
    Choose the THREE most appropriate actions to take in this situation.
    * Option A: Consider how you can learn from this to inform how you behave in future situations
    * Option B: Tell Kathryn that it must be difficult to feel that she is being judged by others
    * Option C: Apologise to Kathryn and Tristan, explaining that you did not mean to cause any offence
    * Option D: Make a joke about the situation in an attempt to calm the situation down
    * Option E: Tell Kathryn that you can ask one of your colleagues to treat her if she would prefer
    * Option F: Explain to Kathryn that if she would like to be treated now, she will need to be treated by you
    * Option G: Ask Kathryn if she would like you to tell your colleagues on the Department that Tristan is her boyfriend to prevent a similar mistake
    * Option H: Explain to Kathryn that you were just trying to ensure that she has the necessary care at home
A

Correct Key: ACH
Rationale: It is important that clinicians have a non-assuming and non-judgemental approach to handling sensitive information during patient contact. It is also important that patients are treated with respect and dignity (General Medical Council [GMC] good medical practice’s Communication, Partnership and Teamwork domain) as well as ensuring that patient feedback and lessons learnt are responded to promptly and appropriately in order to deliver effective and quality healthcare (GMC good medical practice’s Communication, Partnership and Teamwork and the Safety and Quality domains). Therefore, Option A: Consider how you can learn from this to inform how you behave in future situations is the most appropriate action, is an appropriate action to take in this situation. This also serves to reassure the patient and in re-establishing trust in you and the profession (GMC good medical practice’s Communication, Partnership and Teamwork and the Maintaining Trust domains).
It is also important that when things go wrong in practice, doctors act with honesty and integrity as per duty of candour principles by according the wrong, apologise and learn from the experience to inform future practice (GMC good medical practice’s Communication, Partnership and Teamwork and the Maintaining Trust domains). Therefore, Option C: Apologise to Kathryn and Tristan, explaining that you did not mean to cause any offence, would be another action to take in this situation.
As a doctor, it is an essential aspect of good medical practice to ensure that the patient has the information they need and want in a manner that they can understand (GMC good medical practice’s Communication, Partnership and Teamwork domain). Therefore, Option H: Explain to Kathryn that you were just trying to ensure that she has the necessary care at home, would be another appropriate action to take in this situation.

33
Q
  1. You have heard an FY1 colleague, Clare, being shouted at on the ward by a senior doctor regarding an incident during her shift on-call last weekend. Clare is visibly distressed and informs you that she wants to quit medicine.
    Choose the THREE most appropriate actions to take in this situation.
    * Option A: Inform the nurse on the ward that you are taking Clare to a quiet place to calm her down
    * Option B: Report the senior doctor to their supervising consultant
    * Option C: Explain to the senior doctor that they should have handled the situation better to avoid upsetting Clare
    * Option D: Suggest that Clare discusses her concerns with her educational supervisor*
    * Option E: Advise Clare to ignore the senior doctor’s comments
    * Option F: Tell Clare that the senior doctor should have handled the situation better
    * Option G: Advise Clare to carefully consider what other career options are available to her
    * Option H: Advise Clare that she should not quit medicine
A

Correct Key: ACD
Rationale: The first thing to do would be to take your colleague to a quiet place to let her calm down. It is important that you let the nurse co-ordinator of this so you will not be missed, therefore option A would be appropriate action. Option C would also be appropriate as it lets the senior doctor analyse his actions. It may so happen that he may understand that his behaviour was not appropriate and make amends himself. Option D is also an appropriate option as an educational supervisor is there to provide support when needed and could help mediate.
Option B may appear appropriate, but it does nothing to help Clare unless she is willing to engage. It may also detrimentally affect your relationship with your colleagues. Option F and H also do nothing to help the situation and just reiterate something that Clare may already be feeling. Option E is an inappropriate response as Clare is clearly distressed by the comment, which needs to be dealt with appropriately. Advising Clare to consider what other career options are available to her (option G) is also an inappropriate response ,as she is distressed by the incident and your want to calm her down and offer her support. Suggesting Clare carefully considers other career options may cause further upset.

34
Q
  1. You are working on a stroke ward. You missed the start of the consultant ward round, as you were cannulating a patient. At the end of the ward round, the consultant and registrar/specialty trainee* are called urgently to assess a patient. The consultant asks you to organise a CT* scan for Pat, who was the first patient of the ward round. He does not provide any further information before he leaves the ward. When you speak to Pat to explain that you will be organising a CT scan for her, she becomes upset and says that she is fed up of having so many tests.
    Choose the THREE most appropriate actions to take in this situation. * Option A: Explain to Pat that if she doesn’t want to have any of the tests, that is her choice * Option B: Tell Pat that unfortunately, she must have the tests done if she wants to get better * Option C: Empathise with Pat that it must be difficult for her, having to have so many tests * Option D: Explain to Pat that the tests are important, so that the team can give her the best possible care * Option E: Offer to ask one of the nurses on the ward to come and sit with Pat * Option F: Ask Pat if there is anything you can do to help her to feel better * Option G: Ask Pat whether she understands why the CT scan is required and if she has any questions * Option H: Offer to delay further tests until the consultant can come to speak with her
A

Correct Key: CDG
Rationale: Showing empathy is a core clinical skill that a doctor should possess, and option C demonstrates this well. The patient will likely feel that her doctor is prioritising her and is putting themselves in her shoes and allows the establishment of patient rapport. Explaining to Pat the importance of the tests (option D) will emphasise to Pat the reasons behind why it is requested by the consultant, and again emphasises to the patient that we are putting her health first. Option G ensures that we have addressed any concerns or questions that the patient might have and confirms that she understands the rationale behind the scan. This way, we are also beginning to assess her mental capacity in a non-judgemental manner.
Option A (explaining to Pat that she does not have to have to the tests if she does not want to) is not putting the patient’s interest as your priority. No attempts have been made to explore nor understand why the patient is refusing to have further tests. Furthermore, there is no mention of the patient’s mental capacity, nor has this been explored by yourself, so it would be inappropriate to take the patient’s word for it without further exploration.
Option F (asking Pat if there is anything that you can do to help her feel better) is not inappropriate, however is not as good as option C,D or G. Although you may be able to make her feel better, it does not stress to her the reason why she needs the scan. Furthermore, even though she may feel better after you speaking to her, it does not address the issue that she should have the scan to unsure a timely recovery.
Telling Pat that she must have the scan done (option B) is paternalistic and sounds very patronising. It is likely to irritate the patient and make her feel that you are not listening to her. This can exacerbate the situation further and could upset the patient, and even lead to a breakdown in the relationship between doctors and patients.
Option E (offering to ask one of the nurses to sit with Pat) is unlikely going to resolve the patient’s concerns and does not allow for the scan to be organised. It is also taking up the nurse’s time, when he/she may already have a heavy workload and have other jobs that need doing for other patients. Option H (offering to delay further tests until the consultant speaks to the patient) is not ideal as this CT scan may have been requested by the consultant as an urgent scan and delaying further tests could compromise patient care and put the patient at risk of harm.

35
Q
  1. An 80 year old patient, who has been on the ward for three days for treatment of a urinary tract infection, is now much improved and due to be discharged home. The patient is extremely keen to get home, where she lives alone. The patient has two daughters; the first daughter, who is present at the hospital, is keen for her mother to return home. However, her second daughter telephones the ward and leaves a message with one of the nurses to say that she is very unhappy that her mother is being discharged, as she feels that her mother is not coping at home.
    Choose the THREE most appropriate actions to take in this situation.
    * Option A: Continue with the current arrangements for discharge
    * Option B: Try and find out more details about the second daughter’s concerns
    * Option C: Explain to the patient that the second daughter does not want her to be
    * discharged
    * Option D: Contact the second daughter and explain that it is her mother’s right to decide
    * whether she is discharged or not
    * Option E: Ask Occupational Therapy* to assess the patient
    * Option F: Ask the first daughter if she has any concerns about her mother being able to
    * cope at home
    * Option G: Explain to the first daughter that she and her sister need to discuss the issue
    * between them
    * Option H: Ask the patient if she feels that she is able to cope at home
A

Correct Key: BEH
Rationale: The first action here is to explore the daughter’s concerns (option B) to ensure discharge home is safe. In the event of a disagreement the OTs assessment (option E) may alleviate concerns and resolve the matter. The patient’s level of capacity has not been specified, but assuming good capacity, the patient’s wishes must be respected wherever possible (option H).
Continuing with the current arrangements risks damaging the relationship with the family (option A). Although a resolution within the family is desirable (option G) and option (D) is correct, it risks increasing tension within the family and getting involved in the family discussion (option C) may be counterproductive. The first daughter has already stated she is keen for her mother to return home (option F).

36
Q
  1. A five year old patient on your ward, Caleb, has been in hospital for three days with a fever. He has not been eating or drinking normally during this time. Caleb’s diagnosis is yet to be confirmed, despite ongoing investigations. You are asked by the nurse in charge to speak with Caleb’s father, Mr Hamilton, about his son’s progress. Mr Hamilton is angry and displaying aggressive behaviour because Caleb has not yet been diagnosed, and he says that Caleb is not getting any better. The registrar/specialty trainee* is currently in clinic and your consultant has left the ward for the day.
    Choose the THREE most appropriate actions to take in this situation.
    * Option A: Speak to Mr Hamilton, with the nurse in charge present, to try to explore his
    * concerns further
    * Option B: Ask the specialty trainee to speak with Mr Hamilton upon his return from clinic
    * Option C: Ask your consultant to speak to Mr Hamilton the next time she is on the ward
    * Option D: Request that hospital security removes Mr Hamilton from the ward
    * Option E: Inform Mr Hamilton that you will not tolerate his aggressive behaviour
    * Option F: Suggest that Mr Hamilton documents his concerns in writing
    * Option G: Ask the nurse in charge to inform Mr Hamilton that you are still awaiting the
    * results of investigations
    * Option H: Acknowledge Mr Hamilton’s frustrations
A

Correct Key: ABH
Rationale: This question is testing your communication skills in a difficult situation. Whilst in a specialty like Paediatrics, it would be unusual for an F1 to find themselves in this situation, the first priority is to acknowledge Mr Hamilton’s frustration (option H). Not doing this would make it difficult to progress to subsequent steps to ensure resolution. The most appropriate steps here would be a chaperoned conversation with the charge nurse to gather more information on Mr Hamilton’s concerns (option A). As Caleb’s diagnosis is not confirmed, Mr Hamilton may have questions you are unable to answer - therefore conveying these concerns to a senior colleague as soon as they are available (option B) to speak to Mr Hamilton is essential.
It may be appropriate to contact the consultant if the specialty trainee was unavailable (option C), however action needs to be taken on the same day and waiting to speak to the consultant the next time she is on the ward does not address the time sensitivity of the situation. Declining to talk to Mr Hamilton (option G) is likely to make the situation more difficult for a colleague later. At this early stage (option E) and (option F) are likely to be counterproductive and may antagonise Mr Hamilton. (option D) is an inappropriate initial response and is would only be appropriate if you are unable to deescalate the situation or Mr Hamilton was a danger to others on the ward.

37
Q
  1. During a busy night shift, you ask your FY1 colleague, James, to complete a blood test for one of your patients, Mr Rowley. You introduce James to Mr Rowley and shortly after James begins to complete the blood test, and you leave. After waiting a few hours, the blood test results are still not back. You call the laboratory, who state that they have no blood test results for a Mr Rowley, but state that there are results for a different patient on your ward. You have repeated the blood sample for Mr Rowley and sent these to the laboratory. You know that no other patient on your ward has recently had a blood test, so you feel sure that James has labelled the blood test wrong
    Choose the THREE most appropriate actions to take in this situation. * Option A: Inform James that he has labelled Mr Rowley’s blood test wrong * Option B: Encourage James to double check the labelling of blood tests in the future * Option C: Explain to James that the laboratory has stated that they have no blood tests for a Mr Rowley * Option D: Ask James if he remembers labelling Mr Rowley’s blood tests * Option E: Suggest to James that he gets someone to check over his labelling of blood tests in future * Option F: Tell James that you will not ask him to complete blood tests for your patients again * Option G: Inform James that Mr Rowley will not be very happy with him * Option H: Tell James that he has wasted the laboratory’s time
A

Correct key: BCD
Rationale: It is clear from the details in the stem that it is highly likely that James has incorrectly labelled the blood samples. This is a mistake, but it can be easily done and most importantly any mistake is learnt from. You have already resent the samples so the important thing for patient safety has already been done. Therefore, this question is asking about what happens next. Option B, C and D are the most appropriate actions to take.
Option B is an appropriate action to take as by asking James to double check the labelling in future, you are alerting James to the probable error that he has made in a non-judgemental way. Option C is also an appropriate action as you are explaining to James that the laboratory has not received any samples for Mr Rowley, which allows James to come to the same conclusion himself that he had made a mistake and incorrectly labelled the samples. It is factual and non-judgemental, but it is not shying away from explaining to James what has happened. Option D is also an appropriate action, as you are opening channels of communication with James in a non-judgemental way. Asking James if he can remember the details might prompt him to realise the error and address it directly.
Although it is likely that James has incorrectly labelled the samples, option A does not help him address the mistake. Option E is somewhat patronising and not helpful to James and does not address the issue directly. Option F is an over exaggeration to a small mistake, that many professionals will make. Option G may not be true and is likely to upset James unnecessarily. Option H is again unhelpful, as it does not help him to address the mistake and learn from it.

38
Q
  1. You are working on a busy surgical ward with an FY1 colleague, Alberto. Recently, you have noticed that Alberto often leaves tasks unfinished, leaving others to complete them. A nurse on the ward explains that a patient has not yet had an urgent scan that was supposed to be requested by Alberto two hours ago. You discover the scan has not been requested.
    Choose the THREE most appropriate actions to take in this situation. * Option A: Request the scan yourself * Option B: Ask Alberto why the scan was not requested * Option C: Talk to Alberto’s clinical supervisor* about his performance * Option D: Tell Alberto he should apologise to the patient for the delay * Option E: Advise the nurse to discuss this patient with Alberto * Option F: Check if any other urgent tasks are incomplete * Option G: Inform the consultant that Alberto has not requested the scan * Option H: Ask your fellow FY1s on the ward if they have had any similar experiences while working with Alberto
A

Correct Key: ABF Rationale: In this scenario, you have a colleague that is not performing to the level that is expected of them however, the main priority is to ensure that the patient receives his urgent scan. This scan has already been delayed by at least 2 hours therefore, one of the priorities would be to order the scan yourself so that you can make sure that this has been done (option A). Furthermore, it would also be a good idea to check in there are any other urgent tasks pending (option F) to ensure that no one’s care has been compromised. After patient safety and interest has been ensured, then it would be wise to speak to Alberto himself, to find out why the scan was not requested (option B).
Initially, you should speak to the person involved directly to resolve any issues locally, before escalating up. Speaking to the consultant (option G) or the Alberto’s clinical supervisor (option C) bypasses Alberto and does not allow him to address the concerns first. In addition, there may be reasons that he is struggling to complete his tasks, for instance he may find it difficult to use some of the systems to order scans and bloods. If this is the case, then speaking to him directly can allow you to offer support before jumping straight to seniors unnecessarily.
Option H (asking other FY1s on the ward if they have had any similar experiences) is unlikely to be beneficial and is generating gossip within the FY1 cohort, which will upset Alberto if he were to find out. No harm has been done to the patient and you have managed to order the scan now, so there is no absolute need to apologise to the patient and may even upset him/her. Advising the nurse to discuss the patient with Alberto (option E) is likely going to result in further delays, which put the patient at risk of harm.

39
Q
  1. You are working on a respiratory ward. When you arrive on to the ward, you are immediately approached by the nurse in charge. He tells you that a patient has become very unwell in the past five minutes and asks you to immediately see the patient. You check and find that the patient is fine, but she is very confused as to why you looked so concerned when you first entered the room. The nurse in charge has previously approached you in this manner when he thought a patient was unconscious, but when you went to review the patient, she had been asleep.
    Choose the THREE most appropriate actions to take in this situation. * Option A: Ask the nurse to ensure that he is mindful of the urgency of a situation when he is communicating in future * Option B: Reassure the nurse that the patient is fine * Option C: Discuss with the nurse why he thought that the situation was urgent * Option D: Ask the nurse to reflect on his previous errors in judgement * Option E: Apologise for confusing the patient as there is nothing wrong * Option F: Explain to the patient that the nurse in charge told you that she had become unwell * Option G: Ask the nurse to apologise to the patient for causing her distress * Option H: Tell the nurse that he needs to be less dramatic and more accurate when providing updates
A

Correct Key: BCE
Rationale: In this scenario, the nurse was very concerned about the patient and given that after your review you have deemed that the patient is fine, then as part of effective communication within in team, it is important for you to relay this information back to the person who called you in the first place (option B). It is always good practice to keep involved people in the loop as to what is happening and further management steps. This would also be a good opportunity for you to discuss with him why he thought that the situation was urgent (option C) so that you can have a better understanding of the rationale behind his concerns. This discussion will also allow you to identify if he requires further training regarding clinical judgement, and therefore additional help can be offered.
As the patient appears very confused, to reassure the patient, you should also let her know that there is nothing to be concerned about and apologise to her for startling her as such (option E). As you are with the patient already, it is best to relieve her anxiety and confusion whilst you are there with her (option E) rather than asking the nurse to apologise to the patient (option G), which does not add anything significant to the situation. Explaining to the patient that the nurse in charge told you that she had become unwell (option F) is exerting blame onto the nurse and comes across and rather unprofessional. This may also have a negative impact on the patient’s relationship with her nurse. Option A (asking the nurse to ensure that he is mindful of the urgency of a situation when he is communicating in future) and option H (telling the nurse that he needs to be less dramatic and more accurate when providing updates) are not appropriate as they come across as condescending and may deter the nurse from escalating further concerns in the future, in fear of “getting it wrong”. This could have dire consequences of putting patients at risk and should not be encouraged. Asking the nurse to reflect on his previous errors in judgement (option D) is not an unreasonable suggestion but is not the most appropriate action to take in this case.

40
Q
  1. A very ill patient tells you one morning that she wants to discharge herself from the hospital because she has not slept for the last three nights. She says that this is because her bed is next to the nurses’ station and the telephone is always either ringing or the staff are talking on it during the night. She tells you that the staff often seem to be chatting on the telephone rather than discussing medical matters. The patient has not been deemed ready for discharge.
    Choose the THREE most appropriate actions to take in this situation.
    * Option A: Offer the patient some general advice on how to sleep better at night
    * Option B: Ask the patient to tell you more about this situation and her concerns
    * Option C: Explain that telephones are only used at night by nurses for urgent issues that
    * cannot wait until the morning
    * Option D: Document the patient’s concerns in the nursing notes
    * Option E: Inform the nursing staff that the patient is having trouble sleeping because of the
    * noise levels at night
    * Option F: Arrange for the patient to be given some ear plugs
    * Option G: Ask the ward manager about the feasibility of moving the patient to a different
    * bed on the ward
    * Option H: Offer to prescribe a low dose sleeping tablet
A

Correct Key: BEG
Rationale: This question assesses your ability to respond to a concern that has been raised by a patient relating to their experience of care in hospital. Simply hearing more about the situation (option B) will reassure the patient that their concerns are being listened to and may lead to some mutually agreed solutions to the problem. The most practical action to take would be to inform the nursing team (option E) as they may not be aware of the situation and may be able to reduce the amount of noise at night. If the situation cannot be resolved, the simple action of considering moving to another bed (option G) could similarly resolve the situation.
Explaining that phones are only used for urgent issues at night (option C) or providing ear plugs (option F) may be reasonable actions but only attenuate the issue rather than resolving the source of the problem. Documenting the problem in the nursing notes (option D) may be important but won’t resolve the situation. Sedation is associated with a risk of falls in hospital (option H) and is an inappropriate risk to take when conservative measures (e.g. moving beds) can improve the situation.

41
Q
  1. You are on a busy medical rotation. One of your FY1 colleagues confides in you that he is finding the job particularly difficult. He tells you that he feels stressed and that he is not sleeping well. He tells you that he is so tired that he is worried he will make a mistake. He tells you that he is considering taking some time off work to recover and asks for your advice.
    Rank in order the importance of the following considerations in the management of this situation (1= Most important; 5= Least important).
    * Option A: The health and wellbeing of your colleague
    * Option B: The opportunity for financial gain by working extra shifts if he takes time off work
    * Option C: The impact on patient safety if he is working while tired
    * Option D: The impact on the team if he takes time off work
    * Option E: The impact on your workload if he takes time off work
A

Correct Key: CADEB
Rationale: This scenario is about patient focus. Prioritising patient safety and the duty of care is an important part of the professional requirements of the General Medical Council (GMC) regarding the obligation for clinicians to ensure self-care and wellbeing in order to be able to treat others (patients/colleagues) safely, appropriately and effectively. In this scenario, the most important consideration is to ensure that patient safety is prioritised above all else. Therefore, (C) – The impact on patient safety if he is working tired is the most important consideration in this situation.
The health and wellbeing of the colleague is also a priority; therefore the second most important consideration would be (A): The health and wellbeing of your colleague.
As all clinicians work as part of a multi-disciplinary team, the potential impact on others should also be considered as being important, therefore the third most important consideration in this case should be (D): The impact on the team if he takes time off work followed by the impact on you as a colleague (E): The impact on your workload if he takes time off work).
As clinicians, compassion and empathy are considered integral to working well as a team and with others, particularly patients, this requires an altruistic and team-based approach. Therefore, the least important consideration in this case would be financial gain from the absence of the colleague, i.e., (B): The opportunity for financial gain by working extra shifts if he takes time off work.

42
Q
  1. Sandeep, the nurse in charge, informs you that a side room* is needed for a patient who has just been made palliative. Sandeep asks you to speak to a patient, Julia, who is currently in a side room unnecessarily. Sandeep asks you to explain to Julia that she will be moved because the side room is needed for a more severely ill patient. When you tell Julia that she will be moved, she appears concerned, saying “why am I not allowed to be comfortable?”
    Rank in order the appropriateness of the following responses to say in this situation. Assume they are all said in a polite way. (1= Most appropriate; 5= Least appropriate).
    * Option A: “Unfortunately, you do not get to decide which room you stay in”
    * Option B: “We would like to offer a side room to every patient, but there are not enough rooms for every patient to be in a separate room”
    * Option C: “We must prioritise side rooms based on clinical need and you are not considered a priority on this basis”
    * Option D: “I appreciate you are comfortable here, but unfortunately there is a severely ill patient who will benefit from this room even more so than you”
    * Option E: “If you have a problem with the decision you should speak to the nurse in charge”
A

Correct Key: DBCEA
Rationale: This scenario is about effective communication in a difficult situation. Part of the General Medical Council’s (GMC’s) standards for practice include the requirement of clinicians to ensure clear and honest communication. Part of good communication is listening, it is important to acknowledge the patient’s opinion/preference in this case before explaining the need to move the patient. In this case, it is also important to ensure that Julia is treated with respect. This requires compassion, empathy, diplomacy and appropriate assertiveness to ensure that a scarce clinical resource is allocated appropriately based on clinical need. Therefore, (D): “I appreciate you are comfortable here, but unfortunately there is another patient who will benefit from this room even more so than you”, followed by (B): “We would like to offer a side room to every patient, but there are not enough rooms for every patient to be in a separate room”, would be the most appropriate responses based on this premise.
The importance of being honest about resources and how they are allocated is not to be underestimated but as this approach does this by not placing the focus on Julia as a priority, (C): “We must prioritise side rooms based on clinical need and you are not considered a priority on this basis” would be the next most appropriate response.
An inappropriate way to manage this situation would be to alienate Julia rather than working with her to resolve the situation. (E): “If you have a problem with the decision you should speak to the nurse in charge”, suggests that a decision has been made already but that Julia may have another option by referring to the nurse in charge however, it is important to note that this response may be interpreted as Julia’s concerns being ignored.
The least appropriate response is (A): “Unfortunately, you do not get to decide which room you stay in” as this response suggests that Julia has no say in what is happening and rather than fostering a collaborative approach, may make the patient feel powerless and resentful, which in turn form the basis for a poor patient experience resulting in an avoidable formal complaint.

43
Q
  1. You are working on a care of the elderly ward. You are informed that one of your patients was accidentally discharged home yesterday. Upon investigation, you find out that the nursing staff booked transportation for the wrong patient, Mr Rayner, due to an inaccurate handover. The nursing staff realised the error and immediately arranged for the patient to be returned to the ward. Today, during the ward round, Mr Rayner and his son ask you who booked the transportation for him to go home, and demand to know how this mistake happened.
    Rank in order the appropriateness of the following actions in response to this situation (1= Most appropriate; 5= Least appropriate).
    * Option A: Reassure Mr Rayner and his son that a full investigation into this incident will take place
    * Option B: Explain to Mr Rayner and his son that you are unable to disclose who booked the transportation
    * Option C: Explain to Mr Rayner and his son that you will ask that they are kept up to date with any news of the incident
    * Option D: Apologise to Mr Rayner and his son for the incident, explaining mistakes such as this are extremely rare
    * Option E: Offer to ask the nurse in charge of the ward to come and speak to Mr Rayner and his son later
A

Correct Key: DACEB
Rationale: This scenario is about coping with pressure, such as when things go wrong in clinical practice. According to the General Medical Council (GMC), every healthcare professional must be open and honest with patients and people in their care when something that goes wrong with their treatment or care causes, or has the potential to cause, harm or distress. This means that health and care professionals must tell the person (or, where appropriate, their advocate, carer or family) when something has gone wrong, apologise to the person (or, where appropriate, their advocate, carer or family), offer an appropriate remedy or support to put matters right (if possible) and explain fully to the person (or, where appropriate, their advocate, carer or family) the short and long term effects of what has happened. Although the mistake was rectified quickly, it is important to acknowledge the inconvenience, undue stress/anxiety and/or distress that may have been caused to the patient and their family as a result of this mistake. Therefore, the most appropriate response would be, (D): Apologise to Mr Rayner and his son for the incident, explaining mistakes such as this are extremely rare.
Providing reassurance regarding quality assurance and the robust systems in place across the organisation is important to re-establish trust, therefore, the second most appropriate response would be (A): Reassure Mr Rayner and his son that a full investigation into this incident will take place. Maintaining clear channels of communication is integral to any working relationship/partnership, therefore the third most appropriate action would be (C): Explain to Mr Rayner and his son that you will ask that they are kept up to date with any news of the incident. The next most appropriate action would be to provide further details if required regarding the incident as appropriate, therefore, (E): Offer to ask the nurse in charge of the ward to come and speak to Mr Rayner and his son later.
The least appropriate action would be to introduce anything that may be perceived as further barrier(s) to open and honest communication therefore, potentially jeopardising the working relationship between the clinical team and the patient/their family, therefore, (B): Explain to Mr Rayner and his son that you are unable to disclose who booked the transportation would be the last option in this case.

44
Q
  1. You are working with your consultant, Dr Edmund. You have noticed that she works extremely quickly, and you sometimes struggle to keep up with her instructions. While Dr Edmund has gone for a break, a patient, Chris, says, “I didn’t understand anything that Dr Edmund just said. You looked like you didn’t either!” You understood the tasks that Dr Edmund assigned to you to manage Chris, but you are unsure of how she came to some decisions based on his test results.
    Rank in order the appropriateness of the following responses to say in this situation. Assume they are all said in a polite way (1= Most appropriate; 5= Least appropriate).
    * Option A: “I will ask Dr Edmund to come back and answer your questions.”
    * Option B: “I will make sure that I understand all the important details about your management before Dr Edmund leaves.”
    * Option C: “She does speak very quickly, doesn’t she?”
    * Option D: “Is there something in particular you want to discuss about your management?”
    * Option E: “Don’t worry, I am clear what she wants me to do next.”
A

Correct Key: ADBEC
Rationale: This scenario is about patient focus. The General Medical Council (GMC) stipulates that doctors must recognise and work within the limit of their competence. As you are unsure of how the decisions about the patient’s treatment have been made by Dr Edmund, the most appropriate response is (A): “I will ask Dr Edmund to come back and answer your questions.” It is also important to bear in mind that the patient is required to give informed consent to treatment which in this case would not be possible due to a lack of clarity about certain details.
It may be useful to ascertain what the patient’s needs may be and therefore, the next most appropriate response would be (D): “Is there something in particular you want to discuss about your management?
Establishing trust in your abilities increases the chance that the patient will engage appropriately, therefore reassuring the patient with (B): “I will make sure that I understand all the important details about your management before Dr Edmund leaves.” Would be the next appropriate response.
Less reassuring because it suggests that the patient trust in your understanding of what has been said when it is clear there is some uncertainty is (E): “Don’t worry, I am clear what she wants me to do next.”
The least appropriate response is (C): “She does speak very quickly, doesn’t she?” as this may be deemed to be unprofessional because you are criticising a senior colleague and their approach in front of a patient.

45
Q
  1. A 50 year old patient, Mr Cox, is referred by his GP* to the Emergency Department with severe chest pain. You suspect that Mr Cox has acute coronary syndrome. In line with your hospital’s guidelines, you request that the cardiology specialty trainee, Lynda, attends to assess him. However, she refuses as she is too busy.
    Rank in order the appropriateness of the following actions in response to this situation (1= Most appropriate; 5= Least appropriate).
    * Option A: Contact Lynda again, reiterating the urgency for her to assess Mr Cox immediately
    * Option B: Contact the cardiology consultant, asking him to review Mr Cox since Lynda is
    * too busy
    * Option C: Document in Mr Cox’s notes Lynda’s refusal to attend
    * Option D: Contact a senior colleague from your own team, seeking support in managing the situation
    * Option E: Continue managing Mr Cox yourself, following hospital guidelines for managing
    acute coronary syndrome
A

Correct key: DABEC
Rationale: This question focuses on negotiating with a colleague for a patient review in accordance with local guidance on best management of this patient. The most appropriate option is (D), as this patient is critically unwell and you need support to manage both the clinical scenario and the declined referral. Similarly, involvement of a senior colleague may highlight, for example, any issues with the initial referral which led to it being declined. Calling the cardiology specialty trainee back (A) to restate the position may have the desired outcome of getting her to attend, but is less likely to succeed as it has already been tried. Contacting the specialty trainee’s consultant (B) would be reasonable to ensure ongoing patient safety but it does not directly address the issue of dealing with the declined referral and this would usually be an action taken by a senior colleague. In the absence of senior help, continuing to follow the guideline is advisable (E) though it has been established this would still entail expediting the cardiology referral so it would not resolve the situation. Documenting Lynda’s refusal in the notes (C) is important but is not an action which will help you resolve the situation or restore the ongoing safe management of the patient.

46
Q
  1. You are part of the on-call* surgical team in the Emergency Department and are asked to see an acutely unwell 25 year old patient with abdominal pain, who is accompanied by her mother. When you go over to the cubicle to see her, her mother starts talking to you about her daughter’s symptoms. You try to ask the patient some questions but as she attempts to answer, her mother keeps talking over her and answering your questions.
    Rank in order the appropriateness of the following actions in response to this situation (1= Most appropriate; 5= Least appropriate).
    * Option A: Let the mother continue to answer the questions
    * Option B: Explain to the mother that you need the patient to answer the questions
    * Option C: Ask the mother to leave the cubicle for a moment, while you ask the patient some questions
    * Option D: Ask the patient if she wants her mother to remain in the cubicle while you continue to ask her some questions
    * Option E: Inform the mother that she can stay in the cubicle but will need to remain quiet
A

Correct key: BDCEA
Rationale: This scenario is about coping with pressure. It is important to ascertain as much information as possible from the patient in order to inform the assessment and treatment plan. Explaining this to the mother is important for her to understand that although her support is valued, the primary source of information needs to be the patient, therefore, (B): Explain to the mother that you need the patient to answer the questions is the most appropriate response.
Due to the nature of the complaint, you are required to ask very detailed and personal questions which may lead to discussing potentially very sensitive information. In some cases, the presence of the patient’s mother may pose a potential barrier to the patient being able to have an open and honest discussion about their condition. However, it is also important to bear in mind that the patient is relatively young, in pain and may be scared, so the presence of their mother may be a vital source of support/comfort/reassurance for the patient while they are in a vulnerable state. However, it is important to give the patient the choice to make that decision, therefore, the next appropriate response is, (D): Ask the patient if she wants her mother to remain in the cubicle while you continue to ask her some questions.
The next appropriate response is, (C): Ask the mother to leave the cubicle for a moment, while you ask the patient some questions, however, it is important to consider that separating the patient from their mother while they are in a vulnerable state may have a negative impact on the patient. The next appropriate response is, (E): Inform the mother that she can stay in the cubicle but will need to remain quiet. However, it is important to consider that the patient’s choice is not considered as part of this option and this may impede your ability to be able to ascertain the true nature of the condition and therefore the appropriate investigations or treatment if the patient fells restricted in terms of being able to discuss personal or sensitive information in front pf their parent. The least appropriate answer is (A): Let the mother continue to answer the questions because as part of best clinical practice, it is important to ascertain accurate and up to date information from the patient themselves except if they do not have the capacity to do so, e.g., due to mental capacity.

47
Q
  1. You are working on a busy admissions unit where patients are becoming noticeably irritable as they are waiting to be clerked* in. You are about to go on your 30 minute break when the consultant, Dr Lough, asks you to help clerk the remaining patients. Dr Lough acknowledges that you have not had a break yet but says, “if you want to be a successful doctor, you should learn to work without breaks!” You are aware that Dr Lough has not had a break either, however according to the hospital’s policy, you are entitled to a 30 minute break now.
    Rank in order the appropriateness of the following responses to say in this situation. Assume they are all said in a polite way (1= Most appropriate; 5= Least appropriate).
    * Option A: “Dr Lough, I’m sorry I am going on my break now. I’ll be back in 30 minutes.”
    * Option B: “Can I take my 30 minute break first so that I can clerk the patients safely and effectively.”
    * Option C: “I don’t think it is appropriate for you to teach others to avoid taking breaks during their shift.”
    * Option D: “It is important we both take our regulatory breaks throughout the day.”
    * Option E: “If you allow me to take a quick 5minute break, I can then quickly help with the rest of the patients.”
A

Correct Key: BADEC
Rationale: This scenario is about effective communication. Maintaining the health and safety of patients and staff is a priority and it is the clinician’s responsibility to ensure that they are fit and well enough to be able to do so. Working while tired has been linked with avoidable omissions, clinical errors or mistakes in practice. Based on that premise and as long as the patients are clinically stable, (B): “Can I take my 30 minute break first so that I can clerk the patients safely and effectively.” is therefore the most appropriate response.
It is also important to know the policies governing the regulations relating to work hours, therefore the next appropriate response would be, (A): “Dr Lough, I’m sorry I am going on my break now. I’ll be back in 30 minutes.” Followed by (D): “It is important we both take our regulatory breaks throughout the day.” as clinicians also have a duty of care towards their colleagues as well as patients.
The next appropriate response is, (E): “If you allow me to take a quick 5minute break, I can then quickly help with the rest of the patients.” On the surface, the offer to take a shorter break and then rush back to see the patients may seem appropriate, however, it is important to consider that five minutes is not sufficient time to have a sufficient break, therefore not addressing the patient safety element of having a fatigued clinician in charge of patient care.
The least appropriate response on the spot is, (C): “I don’t think it is appropriate for you to teach others to avoid taking breaks during their shift.” however, it may be appropriate to raise this separately if this request becomes frequent one from the consultant.

48
Q
  1. You are in the staff room and you overhear a fellow FY1 colleague, Lara, talking about one of the patients on the ward, who is due to be discharged tomorrow. Lara explains that the patient has invited her out to dinner, which Lara has agreed to. You approach Lara and remind her about her professional boundaries as a doctor, however she tells you it’s “none of your business”.
    Rank in order the appropriateness of the following actions in response to this situation (1= Most appropriate; 5= Least appropriate).
    * Option A: Ask to speak to Lara in private to reiterate that her behaviour is unprofessional
    * Option B: Advise Lara to tell the patient that she cannot go out for dinner with him
    * Option C: Suggest to Lara that she speaks to her other colleagues to see what they think
    * Option D: Discuss the situation directly with Lara’s clinical supervisor*
    * Option E: Ask Lara if she has previously arranged to meet up with patients outside of work
A

Correct Key: ABDCE
Rationale: This scenario is about commitment to professionalism. The General Medical Council (GMC) has clear guidelines about professional boundaries for doctors which outlines the remit for appropriate relationships between doctors and their patients. The GMC standards for practice also stipulate that peers should feedback to each other regarding their practice, therefore it is part of your responsibility as a peer to flag up inappropriate conduct and behaviour but to do this in a professional manner that preserves their privacy and dignity as much as possible. According to these guidelines, it is inappropriate for Lara to fraternise socially with a patient in this manner, therefore the most appropriate action would be, (A): Ask to speak to Lara in private to reiterate that her behaviour is unprofessional.
Based on this premise, the next most appropriate action is, (B): Advise Lara to tell the patient that she cannot go out for dinner with him followed by (D): Discuss the situation directly with Lara’s clinical supervisor and (C): Suggest to Lara that she speaks to her other colleagues to see what they think.
The least appropriate action is, (E): Ask Lara if she has previously arranged to meet up with patients outside of work as this is outside the scope of your interest as a current colleague and therefore not your responsibility to explore further.

49
Q
  1. You have recently started working in the Emergency Department. Today, you are working with a registrar/specialty trainee* who you have not worked with before. At the end of your shift, the registrar/specialty trainee says, “I’ve noticed that you are slow to complete tasks. You need to work harder” and then walks away. This is the first time that you have received feedback like this and you think that you have been doing well since starting in the Emergency Department.
    Rank in order the appropriateness of the following actions in response to this situation (1= Most appropriate; 5= Least appropriate).
    * Option A: Ask some of the other doctors working in the Emergency Department if they think you are slow
    * Option B: Discuss the comment made by the registrar/specialty trainee with your clinical supervisor*
    * Option C: Arrange to speak to the registrar/specialty trainee to discuss her feedback in more detail
    * Option D: Try to increase the number of patients you see in a shift
    * Option E: Compare the number of patients you saw on a shift to the other FY1 doctors working in the department
A

Correct key: CBAED
Rationale: This scenario is about coping with pressure. According to the General Medical Council’s (GMC’s) standards of practice, it is your responsibility as a doctor to seek feedback on your practice as part of your ongoing development as a clinician. Therefore, the most appropriate action in this case is, (C): Arrange to speak to the registrar/specialty trainee to discuss her feedback in more detail.
The next course of action would be to seek further guidance or support from your designated clinical supervisor who is responsible for facilitating your development, therefore the next appropriate course of action is, (B): Discuss the comment made by the registrar/specialty trainee with your clinical supervisor.
Peer review and peer feedback is an integral part of your development as a clinician. It also enables you to benchmark your performance against the expected norm for your grade and specialism, therefore it would be beneficial to gain a broader understanding of your performance by seeking feedback from those working in the same setting to put the registrar’s feedback into an appropriate context for you. Therefore, the next appropriate course of action is, (A): Ask some of the other doctors working in the Emergency Department if they think you are slow followed by (E): Compare the number of patients you saw on a shift to the other FY1 doctors working in the department.
The least appropriate course of action in response to the feedback is, (D): Try to increase the number of patients you see in a shift as it is important to establish the accuracy, appropriateness and relevance of the feedback to you and your development in the first place before developing a course of action in response to it as this option may be detrimental to the quality of care you are able to deliver in this setting.

50
Q
  1. An FY1 colleague is working on a very busy ward during his first placement. He regularly misses the foundation teaching sessions* which are organised for FY1s by his clinical supervisor, as he says he is too busy. It is expected that FY1s attend. He tells you that he is concerned that he has missed some of the teaching sessions, but that he feels his experience from working on the ward is valuable. You have found the teaching sessions very useful so far.
    Rank in order the appropriateness of the following responses to say in this situation. Assume they are all said in a polite way (1= Most appropriate; 5= Least appropriate).
    * Option A: “It is up to you to decide how you prioritise your time.”
    * Option B: “Both the teaching sessions and ward experience will be useful to develop your knowledge.”
    * Option C: “I don’t think that the ward experience alone will give you all of the clinical knowledge you will need.”
    * Option D: “Have you thought about discussing this issue with your clinical supervisor
    ?”
    * Option E: “Would it be helpful for me to explain what I find useful about the teaching sessions?”
A

Correct Key: DBECA
Rationale: This scenario is about commitment to professionalism. The General Medical Council (GMC) standards of practice and guidelines for good practice stipulate that doctors should actively engage in Continuous Professional Development (CPD) and take advantage of any opportunities provided to advance their development as a clinician. Therefore, ensuring attendance at the foundation teaching sessions is essential to his development. The most appropriate response to this situation is, (D): “Have you thought about discussing this issue with your clinical supervisor*?” as the FY1’s clinical supervisor will be able to support him in terms of caseload management, time management, prioritisation, etc., to facilitate the FY1 to attend the teaching sessions. Although the practical ward experiences are valuable, the FY teaching sessions are also valuable and essential for new clinicians to consolidate their learning/development. Sharing your personal experiences of the benefits of these teaching sessions may also encourage the FY1 to explore avenues to prioritise attendance to the FY teaching sessions.
Therefore, the next appropriate response is, (B): “Both the teaching sessions and ward experience will be useful to develop your knowledge.” followed by (E): “Would it be helpful for me to explain what I find useful about the teaching sessions?” as the additional details of the benefits of the sessions may be useful for your FY1 colleague to better understand the relevance of these sessions to his developmental journey.
Peer review and peer feedback is an essential aspect of your development as a doctor. It is also a professional requirement outlined by the GMC for all grades, therefore, (C): “I don’t think that the ward experience alone will give you all of the clinical knowledge you will need.” particularly when it comes to sharing your experiential learning is a professional responsibility.
The least appropriate response is, (A): “It is up to you to decide how you prioritise your time.” as this advice may wrongly encourage the FY1 to continue to miss out from essential teaching for new clinicians, resulting in deficits (knowledge and skills) that potentially may be detrimental to his development and practice. This response is also inconsistent with developing self and supporting others to develop which is part of the NHS knowledge and skills framework, it is also not aligned with the concepts of candour, honesty and probity, all professional values integral to the medical profession.

51
Q
  1. You are looking after a patient, Roisin, on the Cardiology ward when she invites you to attend her husband’s birthday party at her home next weekend. You have been on the medical team that has been looking after Roisin for the past two months, and she is due to be discharged tomorrow. You do not have any plans for next weekend; however, you are aware that it is inappropriate to see a patient socially outside of work.
    Rank in order the appropriateness of the following actions in response to this situation (1= Most appropriate; 5= Least appropriate).
    * Option A: Thank Roisin for the invitation but explain that it would not be appropriate for you
    to attend
    * Option B: Invite other individuals from the medical team that has looked after Roisin to
    attend the party with you
    * Option C: Politely decline Roisin’s invitation, saying that you cannot attend
    * Option D: Ask another doctor on the ward to manage Roisin’s care until she is discharged
    * Option E: Explain that you are unable to attend, but buy Roisin a small gift
A

Correct Key: ACDEB
Rationale: This question explores professional boundaries between a doctor and their patient. The most appropriate action is to maintain an appropriate professional boundary and give Roisin an explanation as to why you cannot attend (A). Declining to attend without an explanation (C) would maintain the required boundary, but without an explanation it may upset Roisin or cause her to repeat the request at a later time. Asking another doctor on the ward to manage Roisin’s care until she is discharged does not adequately address the invitation (D) and may similarly cause her to repeat her request. Explaining you are unable to attend without explanation and buying a gift (E) conveys a mixed message and would most likely be viewed as inappropriate. It is clearly not appropriate to attend the party (B). Inviting other members of the healthcare team compounds the blurring of professional boundaries.

52
Q
  1. You are working in the Emergency Department. While walking through the waiting room, a patient approaches you. She says “When are you going to see me? I have been waiting for hours! Do you not understand that I have other things to do today? I cannot stand this waiting room either, it is so noisy in here. This place is a joke!”. Her voice is raised, and she is clearly very frustrated. This patient is not being prioritised because her condition does not require urgent treatment.
    Rank in order the appropriateness of the following responses to say in this situation. Assume they are all said in a polite way (1= Most appropriate; 5= Least appropriate).
    * Option A: “Please continue to wait patiently as the team are working as quickly as they can.”
    * Option B: “Unfortunately, we have to prioritise patients based on their condition, which is why some patients have to wait longer than others.”
    * Option C: “You do not require urgent treatment, so please continue to wait.”
    * Option D: “It is unreasonable for you to shout in the waiting room. Unfortunately, you have to wait to be seen.”
    * Option E: “I understand it can be frustrating to wait, but rest assured the team are working hard to see people as quickly as they can”
A

Correct key: EBADC
Rationale: This scenario is about effective communication. The General Medical Council (GMC) prioritises effective communication as a key skill for clinicians. As part of the professional requirements for the medical profession, you are expected to communicate clearly and honestly with patients and their families and to ensure a compassionate, empathetic, and respectful approach. Therefore, the most appropriate response to address this situation is, (E): “I understand it can be frustrating to wait, but rest assured the team are working hard to see people as quickly as they can” as this response not only acknowledges and validates the patient’s negative experience but demonstrates empathy about how the situation may be affecting the patient and others. In addition, this response reassures the patient of non-discrimination as the team is working hard to see people as quickly as possible.
Duty of candour is the professional requirements for honesty, particularly when things are not going well or have gone wrong in practice. It also provides the patient with a different perspective on why things have happened, therefore, the next appropriate response is, (B): “Unfortunately, we have to prioritise patients based on their condition, which is why some patients have to wait longer than others.” as this enables you to explain the reason for the delay in the patient being seen in an honest manner.
The next appropriate response is, (A): “Please continue to wait patiently as the team are working as quickly as they can.” as this response does not explain the situation to the patient but politely asks for them to be patient. However, it is important to consider that providing the context for the patient may encourage the patient to be more compliant.
The NHS has a zero tolerance to all forms of abuse (verbal or physical) against its staff, including aggression. It is important that inappropriate conduct is addressed in the early stages to prevent escalation of negative behaviour. However, it is important to consider that this approach may not be the ideal approach to de-escalate this situation. Therefore, addressing the patient’s behaviour as a response to their complaint using (D): “It is unreasonable for you to shout in the waiting room. Unfortunately, you have to wait to be seen.” is the fourth most appropriate response to this situation.
The least appropriate response in this situation is (C): “You do not require urgent treatment, so please continue to wait.” as this response fails to address any of the patient’s complaint. Therefore, it may be deemed as being unprofessional as it fails to demonstrate the positive professional values such as care, compassion, communication, etc., which are core requirements of the profession. In addition, the response may be interpreted by the patient as their needs/concerns are not important or relevant, leading to a negative patient experience which may escalate to an avoidable formal complaint.

53
Q
  1. Tonight, you are working on-call* in general surgery, which includes covering the Paediatric ward. A nurse tells you that the registrar/specialty trainee* has requested that you to insert a cannula on a 12 year old patient, while he is in theatre operating on another patient. You have never been given training on how to cannulate children.
    Rank in order the appropriateness of the following actions in response to this situation (1= Most appropriate; 5= Least appropriate).
    * Option A: Wait for the registrar/specialty trainee to finish in theatre so he can show you what to do
    * Option B: Watch an instructional video on how to cannulate children, before inserting the cannula
    * Option C: Phone the registrar/specialty trainee whilst he is in theatre to explain that you cannot insert the cannula
    * Option D: Try to cannulate the patient as instructed by the registrar/specialty trainee
    * Option E: Establish the urgency of inserting the cannula before deciding what to do
A

Correct key: ECABD
Rationale: This scenario is about coping with pressure. The General Medical Council (GMC) stipulates that, doctors must only practice within their scope of competence. This is essential to maintain safety in clinical practice and ensures that patients receive appropriate and effective care. In all circumstances, it is important to assess the situation before developing a plan of action, therefore the most appropriate course of action is, (E): Establish the urgency of inserting the cannula before deciding what to do.
According to the GMC seeking support/help/guidance is a professional requirement for the medical profession, therefore, the next appropriate course of action is, (C): Phone the registrar/specialty trainee whilst he is in theatre to explain that you cannot insert the cannula.
If the insertion of the cannula is not urgent, the next appropriate action is, (A): Wait for the registrar/specialty trainee to finish in theatre so he can show you what to do.
The next appropriate course of action is, (B): Watch an instructional video on how to cannulate children, before inserting the cannula. The GMC professional requirements for doctors includes a competency framework in which all clinical skills must be assessed and the clinician deemed to be competent and safe by an appropriately trained/qualified person before they can perform the skill independently/unsupervised on a patient. Operating outside the competency framework by using an instructional video before carrying out cannulation process on the patient is dangerous practice, unprofessional and negligent.
The least appropriate course of action in this situation is (D): Try to cannulate the patient as instructed by the registrar/specialty trainee as this involves a doctor practicing blindly outside their scope of competence. Acting outside the scope of competence is unacceptable as it is negligent and is in direct contradiction of the professional requirements and expectations of the medical profession.

54
Q
  1. You overhear the radiologist shouting at your FY1 colleague about a CT scan* request. The radiologist says rude remarks about the FY1’s intelligence and refuses to do the scan, leaving your FY1 colleague visibly upset. You go and speak to your FY1 colleague, who explains that the consultant had asked him to request the CT scan. When you suggest reporting the radiologist’s rude behaviour, your FY1 colleague ask you not to mention this to anyone. You have heard from other members of staff that this radiologist has spoken rudely to them before.
    Rank in order the importance of the following considerations in the management of this situation (1= Most important; 5= Least important).
    * Option A: That your FY1 colleague has requested not to mention it to anyone
    * Option B: The impact of the radiologist’s behaviour on patient care
    * Option C: That the radiologist is more senior than your FY1 colleague
    * Option D: The radiologist’s past behaviour with other members of staff
    * Option E: That you were not directly involved in the incident
A

Correct key: BDACE
Rationale: This scenario is about coping with pressure. Research indicates that in most cases, poor conduct is usually not isolated to one aspect of a professional’s job role. It is therefore important to consider other aspects of that may be affected by the radiologist’s poor conduct, the first priority being patient safety and patient care. Therefore, the most important consideration in the management of this situation is, (B): The impact of the radiologist’s behaviour on patient care.
Also of significant importance is the impact on other staff and the multi-disciplinary team, therefore the next important consideration is, (D): The radiologist’s past behaviour with other members of staff as this may have detrimental impact on others as well as their ability to deliver effective patient care.
The next important consideration is, (A): That your FY1 colleague has requested not to mention it to anyone. However, it is important to note that your colleague’s request is superseded by the duty of care to patients and colleagues and the expectation/professional responsibility as a doctor to flag up concerns in practice that have the potential to adversely impact on patients or staff health and wellbeing.
The next important consideration is, (C): That the radiologist is more senior than your FY1 colleague as all healthcare professionals regardless of grade have a professional code of conduct to adhere to as stipulated by the relevant professional statutory body, e.g., the GMC for doctors and the Health and Care Professions Council (HCPC) for Allied Health Professionals (AHPs) such as radiologists. There are also core professional requirements including communication, professionalism, teamwork and competency that apply to all healthcare professionals regardless of their grade. This ensures that grade or rank should not be an excuse for poor professional conduct in clinical practice. The NHS Leadership Academy outlines the importance of senior staff across the organisation demonstrate leadership traits and are accountable for their conduct. The more senior grade of the radiologist means that there is an expectation that they lead by example by demonstrating positive professional values, not only to facilitate better cohesiveness across the organisation and better teamwork but so that more junior clinicians may aspire to higher levels of professionalism. The least important consideration is, (E): That you were not directly involved in the incident as this does not absolve you of your duty of care to patients and colleagues or your professional responsibility in terms of flagging up concerns in practice.

55
Q
  1. You are the FY1 on-call* overnight. You are bleeped* by one of the nurses about non-urgent ward tasks. You explain that you are busy with a sick patient, but you will attend when you have completed your urgent tasks. An hour later you are bleeped twice within 5 minutes by the same nurse. When you do not respond, the nurse then calls you, regarding the same non-urgent ward tasks, asking when you are coming.
    Rank in order the appropriateness of the following responses to say in this situation. Assume they are all said in a polite way (1= Most appropriate; 5= Least appropriate).
    * Option A: “Please don’t bleep me again about non-urgent matters.”
    * Option B: “If you bleep again, I will ignore it and not complete the tasks.”
    * Option C: “I will attend to non-urgent tasks, but sick patients always take first priority.”
    * Option D: “If I don’t answer a bleep, please assume that I am with a sick patient and leave some time before trying again.”
    * Option E: “By constantly bleeping me, you are delaying me from helping you.”
A

Correct key: CDEAB
Rationale: This scenario is about effective communication. As part of a multi-disciplinary team, it is not unusual that different members of the team may have conflicting priorities. In order for the team to work well together, maintaining effective communication is key. Therefore, the most appropriate response to say in this situation is, (C): “I will attend to non-urgent tasks, but sick patients always take first priority.” as this explains the rationale for your decision to delay attending to her request to your nurse colleague. Therefore, promoting better understanding about the challenges other members of the team may be facing which promotes better teamwork.
As part of establishing a good working relationship with others, it is important to be clear about routine practice to set expectations, therefore, (D): “If I don’t answer a bleep, please assume that I am with a sick patient and leave some time before trying again.” is the next most appropriate response. This response enables the ward staff and nursing colleagues to understand how you work as well as the challenges you face as part of your role which they may be unaware. This response will also help outline what to expect from you as their FY colleague on call on the night shift, which will make for better teamwork in the future.
The next most appropriate response is, (E): “By constantly bleeping me, you are delaying me from helping you.” as this explains the impact of her continuous actions on you and others, potentially enabling her to be more mindful about the challenges other colleagues may be facing as part of their roles in the team and her conduct when contacting the FY on all in the future.
Although not an ideal approach for fostering working relations, it is sometimes important to establish firm boundaries with some members of the team. Therefore, (A): “Please don’t bleep me again about non-urgent matters.” is the next appropriate response, particularly in the face of the nurse’s continuous actions over the course of the night shift. In the long term, this may improve working relationships between you as there will be a clear understanding if what is/is not appropriate.
The least appropriate response is, (B): “If you bleep again, I will ignore it and not complete the tasks.” as this is an inappropriate and unprofessional way to respond to a colleague. It is also unprofessional not to respond to a work-related call from a colleague or not to complete clinical tasks allocated to be addressed on the night shift as this may be deemed as neglectful and dereliction of your duty to support the delivery of effective patient care on the night shift. Such an action may result in poor care being delivered or delay in care being delivered which may result in poor health outcomes for patients, such as deterioration in health status.

56
Q
  1. This is your first post on a busy surgical rotation, and you are struggling with your workload. You have been assigned a combined clinical and educational supervisor, Mr Jones, for your placement. So far you have emailed Mr Jones twice, with no response, and your initial induction meeting was due two months ago.
    Rank in order the appropriateness of the following actions in response to this situation (1= Most appropriate; 5= Least appropriate).
    * Option A: Refrain from attempting to make contact with Mr Jones, waiting for him to contact you * Option B: Report to the Foundation Programme Director
    the lack of response from Mr Jones * Option C: Contact Mr Jones’ secretary, in order to find Mr Jones and discuss your induction meeting * Option D: Ask the Foundation Programme Director to assign you a new educational supervisor * Option E: Ask the hospital switchboard* to transfer your call to Mr Jones’ mobile telephone * number
A

Correct Key: CBEDA
Rationale: This scenario is asking you to rank the appropriateness of a series of steps to ensure you receive and are able to evidence your essential supervision meetings as an FY1 doctor. Whilst the question does not specify why Mr. Jones has not been in contact, attempting to reach him through his secretary (C) is the most appropriate initial step. Highlighting your difficulty to the Foundation Programme director is also important (B) - as they may be able to support you in obtaining a response and they need to be aware of the issue. The Foundation Programme Director’s awareness of the problem and the steps you have taken to resolve it will prevent the delayed meeting reflecting badly on you, which could potentially impact on your progression through the foundation programme. Option (E) is an alternate method of contacting your supervisor but is likely to be less appropriate than seeking information through Mr Jones secretary of when he might be available. Similarly, requesting an alternate educational supervisor (D) is not unreasonable but is a final resort if a meeting cannot be arranged with Mr Jones with the support of the Foundation Programme Director. Taking no action here (A) is not an appropriate response as there is a risk that no meeting will occur at all. This would reflect poorly on you and is likely to have an impact on your progression through training.

57
Q
  1. You arrive home one evening after finishing a shift at the hospital, when you remember that you have forgotten to handover some important information at the handover meeting. A patient requires an urgent blood sample which needs to be taken today. You are back at work tomorrow morning.
    Rank in order the appropriateness of the following actions in response to this situation (1= Most appropriate; 5= Least appropriate).
    * Option A: Take the blood sample when you return to work tomorrow morning * Option B: Return to the ward immediately to take the blood sample * Option C: Send a text message to the doctor who is currently covering the ward * Option D: Contact the doctor covering the ward, to explain the situation * Option E: Phone the ward directly and leave a message with the nursing team
A

Correct key: DECBA
Rationale: This scenario is about commitment to professionalism. In view of the heavy workload and sometimes extremely challenging circumstances that you work in healthcare, it is not unexpected that at times, you may forget something, this is part of being human. In this case, the most appropriate and professionally responsible course of action is to ensure that the patient gets the care they need as soon as possible in order to minimise the impact of the delay in care on their recovery. Therefore, (D): Contact the doctor covering the ward, to explain the situation is the most appropriate action in response to this situation. The next most appropriate course of action, particularly if unable to liaise directly with the doctor covering the ward, would be to inform another member of the clinical team. Therefore, (E): Phone the ward directly and leave a message with the nursing team in a concerted effort to ensure that the patient gets the care that they need as soon as possible. The next most appropriate action in this situation is, (C): Send a text message to the doctor who is currently covering the ward. However, it is important to note that without an acknowledgement that the message has been received, read, understood and actioned, there is a risk that the text message may be overlooked, resulting in the patient not getting the important blood test that they need for their care to be optimised. At the end of a shift when a clinician is probably very tired, there is a higher risk of error/mistake occurring because of human factors. Therefore, (B): Return to the ward immediately to take the blood sample is not an ideal action to consider as a solution in this situation, particularly when there are other clinical staff present on site who are able to carry out the same task. The least appropriate course of action in this situation would be to do nothing/take no action. Therefore, (A): Take the blood sample when you return to work tomorrow morning is the least appropriate course of action to take as it will delay the care the patient requires which may have a detrimental impact on the patient’s health. This course of action is unprofessional, irresponsible and dangerous practice which is in contradiction to the expectation and professional requirements for the medical professional. This conduct is also not aligned to the four pillars of medical ethics which include, beneficence and non-maleficence when making decisions relating to patient care.

58
Q
  1. You have had a busy day at work, and you are due to finish work soon. A senior doctor approaches you to tell you that the doctor due to start the night shift has called in sick. She asks you if you will stay an extra three hours until another doctor is available to cover the rest of the shift. You have plans to meet a friend for dinner after work.
    Rank in order the importance of the following considerations in the management of this situation (1= Most important; 5= Least important).
    * Option A: The length of the shift you have just worked
    * Option B: The frequency of you being asked to work extra hours
    * Option C: Whether there are other doctors who could stay for another three hours
    * Option D: How much you will get paid for the extra work
    * Option E: Your social life commitments for the evening
A

Correct Key: ACBED Rationale: This question related to providing patient care in unexpected emergency situations within the working time directives. The most important consideration is the number of hours you have already worked in your own shift (A) and whether doing more hours will breach the working time directives which are in place for patient safety. It is also very important to consider if there are other doctors who could stay without breaching the working time directive (B) as this will also allow you to continue with your pre-arranged commitments. While it is important to consider how often this situation arises in the workplace (C), as it may highlight an underlying deficiency in workforce planning, it should not affect your ability to help on this particular occasion in the interest of patient care. Your social life commitments are important for your wellbeing, but it is expected that patients’ needs will be given priority in emergency situations. The amount of financial compensation (D) is the least important consideration in your ability to provide staffing cover in an emergency situation if it is safe to do so.

59
Q
  1. You are working on a medical ward. This morning, your consultant asked you to request a renal review for a patient. You return from your lunch break to find the registrar/specialty trainee* has just finished reviewing a different patient on the ward. You realise that you accidentally referred the wrong patient, who had a similar name, to the renal registrar/specialty trainee. You inform the renal registrar/specialty trainee of your mistake, and he becomes annoyed and says that you have wasted an hour of his time.
    Rank in order the appropriateness of the following responses to say in this situation. Assume they are all said in a polite way (1= Most appropriate; 5= Least appropriate).
    * Option A: “That was a silly mistake. We all make mistakes though, don’t we?”
    * Option B: “Sorry I know that was my fault, but do you have any time to review the correct patient now?”
    * Option C: “Sorry, I know that this was my mistake, but I think we should focus on the correct patient having a renal review.”
    * Option D: “I thought you would double check before spending an hour reviewing the patient.”
    * Option E: “Please can you review the correct patient, it should only take a few minutes.”
A

Correct Key: BCEAD
Rationale: The most appropriate response here would be to acknowledge and take appropriate steps to ensure that patient receives necessary care. It is also important to acknowledge how other people may react in this situation. Therefore, B, (requesting a review) is the most appropriate and professional response followed by suggesting a review (C). Option E and A are less appropriate responses as they do not consider the impact of your mistake on the workload of other colleagues. Although you may expect that patient identity should be verified by professionals it is unlikely to be helpful if wrong patient details were given. Passing the responsibility to a colleague (D) is an inappropriate response as it is your responsibility to provide appropriate patient care to your patients.

60
Q
  1. You are walking along the corridor when Simon, your registrar/specialty trainee*, approaches you and asks you to assist him in reviewing a sick patient. When you arrive, the nurse takes you to one side and she tells you that he thinks Simon has been drinking. You can tell that he smells of alcohol. The review is urgent and Simon is needed as you are unable to conduct the review by yourself.
    Rank in order the appropriateness of the following actions in response to this situation (1= Most appropriate; 5= Least appropriate).
    * Option A: Suggest to the nurse that she asks Simon whether he has been drinking * Option B: Explain to Simon that he should not conduct the review if he is under the * influence of alcohol * Option C: Contact the consultant immediately to explain the situation * Option D: Ask a specialty trainee from another ward to review the patient with you * Option E: Continue with the review, but explain to Simon afterwards that he smells of * alcohol
A

Correct Key: BDCAE
Rationale: Your initial action is to safeguard the patient, asking your specialty trainee to refrain from any interaction if under the influence of alcohol (B) is appropriate as Simon is directly working with the patient at this time and it is important to safeguard them. It is possible there is an alternative explanation and this allows Simon to take suitable action himself. You should seek help from another specialty trainee to avoid delaying the patient’s review (D) and inform the consultant (C) after prioritising the patient’s needs. Delegating the difficult conversation to your nursing colleague is not appropriate unless the nurse is very senior and has a good working relationship with Simon (A). You should not continue with the review with Simon as this could put the patient at risk (E), which would be the least appropriate response.

61
Q
  1. You are just about to leave for your foundation teaching session* when a nurse asks you to speak to a patient’s relatives, who have asked for an update from a doctor on his progress. You have recently missed some teaching sessions due to night shifts and annual leave. You know you need to attend this session to reach the minimum amount of teaching to pass the Annual Review of Competence Progression (ARCP)*.
    Rank in order the appropriateness of the following actions in response to this situation (1= Most appropriate; 5= Least appropriate).
    * Option A: Ask the patient and his relatives if you can discuss his progress when you are back from the teaching session
    * Option B: Ask one of your colleagues if they would be willing to have a discussion with the patient and his relatives
    * Option C: Miss the teaching session so that you can discuss the patient’s progress with him and his relatives
    * Option D: Discuss the patient’s progress with him and his relatives and attend the teaching session late
    * Option E: Inform the nurse that you are busy and continue on your way to the teaching session
A

Correct Key: ABDCE
Rationale: This scenario highlights the need to balancing patient needs with your own training requirements both of which are important. It is expected that the workforce arrangements will have adequate provision rota for you to be released to attend your teaching and training. (A) is the most appropriate option as you are taking responsibility for the task but seeking patient and relative’s permission to assess the urgency of the task and seeking their engagement to allow you to attend teaching. Next best option is to seek help from a colleague (B) which allows you to attend your training and address patient’s need. Missing the teaching partially (D) or completely (C) are not appropriate actions unless it is the last resort. (E) is the least appropriate response as it does not take patient care into account and may be considered unprofessional.

62
Q
  1. You are a FY1 doctor working on a respiratory ward. Last week, you completed a ward round with your consultant and saw a patient with advanced dementia, who was verbally abusive towards you. You found this interaction extremely challenging. Today, your consultant asks you to do a review of the patient by yourself. The patient is recovering from a chest infection and is by himself in a side room of the ward. You explain to your consultant that you found the last interaction with the patient very difficult, however, the consultant says, “all you need to do is examine the patient’s chest. You will be OK.”
    Rank in order the appropriateness of the following actions in response to this situation (1= Most appropriate; 5= Least appropriate).
    * Option A: Request that the consultant attends the examination of the patient with you
    * Option B: Ask your FY1 colleague if she is available to help you examine the patient
    * Option C: Ask the consultant if there are any strategies that you could use to manage the patient’s abusive behaviour
    * Option D: Inform the consultant that you do not want to conduct the examination following your last experience with the patient
    * Option E: Explain to the consultant why you found the interaction with the patient last week extremely challenging
A

Correct key: ECBAD
Rationale: This scenario is about coping with pressure. The General Medical Council (GMC) stipulates that, doctors must recognise and work within the limit of their competence. As part of your ongoing development, it is important to be honest about limitations and to seek advice and guidance to support your development. Demonstrating self-awareness through reflective practice to identify specific challenges and deficits you may have is essential to your development as a clinician. Therefore, (E): Explain to the consultant why you found the interaction with the patient last week extremely challenging would be the most appropriate response to your consultant’s request to make them aware that this is an area you need further support on.
The second most appropriate response is (C): Ask the consultant if there are any strategies that you could use to manage the patient’s abusive behaviour as this will enable the consultant to provide appropriate guidance on practical patient handling and communication skills to enable you to be able to examine the patient effectively. These skills are invaluable and transferable skills to add to your professional portfolio. Although peer support and collaborate work is recommended to support your development, it is important to bear in mind that in complex cases such as this, a fellow FY colleague may not have the prerequisite knowledge, skills or expertise to help you examine the patient.
It is also important to be mindful of the nature of the patient’s condition, specifically in terms of how the presence of two strangers trying to examine him may affect a patient with advanced dementia, e.g., cause distress, agitation, etc., therefore, (B): Ask your FY1 colleague if she is available to help you examine the patient is the third most appropriate response.
The fourth most appropriate response is (A): Request that the consultant attends the examination of the patient with you because although the patient’s condition and presentation may be complex, the task that you are being asked to carry out by your consultant is a basic one, i.e., a chest examination and therefore a task that an FY on a respiratory ward would be expected to be able to carry out independently. However, it must be acknowledged that is the complex presentation is presenting a significant limitation to being able to carry out the task, it is appropriate to seek senior support and guidance.
The least appropriate response is (D): Inform the consultant that you do not want to conduct the examination following your last experience with the patient as it may be perceived as unprofessional and it demonstrates a lack of compassion, poor understanding and insight into the signs/symptoms of the patient’s chronic condition (dementia) and how other factors such as sepsis/infection may affect or impact on the patient. This response may be perceived as dereliction of duty, i.e., a clinician failing in their duty of care for a vulnerable patient.

63
Q
  1. You are working on an obstetrics and gynaecology ward. Your fellow FY1 colleague tells you that he is about to review a 20 year old patient who has attended for a termination of pregnancy. Your FY1 colleague tells you that it makes him angry that anyone would choose to have a termination and is going to try and persuade the patient to not have the termination.
    Rank in order the appropriateness of the following responses to say in this situation. Assume they are all said in a polite way (1= Most appropriate; 5= Least appropriate).
    * Option A: “You shouldn’t let your feelings about termination be known to the patient.”
    * Option B: “You’re entitled to your own opinion.”
    * Option C: “It is unprofessional to be judgemental towards others who have different beliefs to you.”
    * Option D: “Some terminations are not due to the choice of the patient, it could be medically indicated.”
    * Option E: “You should respect the patient’s decision regardless of your beliefs.”
A

Correct Key: ECADB
Rationale: It is the right of every patient to be able to access healthcare without discrimination. It is also important that personal beliefs do not become a barrier to effective patient care, therefore option E is the most appropriate response as it reminds your colleague of this. Next would be option C, as it again reiterates that it is the patient’s choice and it is unprofessional to be judgemental, although you have made an assumption about your colleagues differing beliefs to that of the patient. Letting a patient know your feelings can be construed as an attempt to influence their decision, so option A would come next, but may be considered confrontational by your colleague. Option D appears to defend the patient’s decision and you may think that it is appropriate. However, it should never come to that as the reason for the termination is of no consequence and the patient’s wishes should be respected irrespective of the reason. Option B is the least appropriate as it does nothing to ensure that the patient is being respected or provided with appropriate care.

64
Q
  1. You have started your shift in a busy hospital ward but one of your colleagues, Harriet, is late. Harriet has been late every day this week and this has increased your workload. As she is not there, your consultant, Dr Grey, asks you to review one of her patients, Adeel. You do not know Adeel’s history and are concerned that this might result in you making a mistake. Dr Grey tells you that the ward is short staffed and asks you to do the review as quickly as you can.
    Rank in order the appropriateness of the following actions in response to this situation. (1= Most appropriate; 5= Least appropriate).
    * Option A: Explain to Dr Grey that Adeel is not your patient so you do not feel comfortable conducting the review
    * Option B: Inform Dr Grey that you may need to ask him some questions after Adeel’s review to check your understanding
    * Option C: Suggest to Dr Grey that you will complete a very basic review of Adeel
    * Option D: Ask Dr Grey to supervise the review as Adeel is not your patient
    * Option E: Ask Dr Grey for any further background information that might help you understand Adeel’s presentation
A

Correct Key: EBDAC
Rationale: This scenario is trying to test our ability to cope with pressure without compromising patient safety. Since you do not know Adeel’s medical background or problems it is most appropriate action to seek this information beforehand (E) so that your review is appropriate and efficient. The next best option is to do a review first and then seek clarification on issues which are not clear (B) followed by asking Dr Grey to supervise the review (D) which is safe practice but may not be as efficient in the given circumstances. Reluctance to conduct a review (A) or conducting a basic review (C) are not desirable options in this situation. Between option A and C sharing your reluctance to conduct the review is more appropriate than performing an inadequate review, therefore option C would be least appropriate.

65
Q
  1. Your FY1 colleague, Samira, looks unwell during the ward round. When you ask how she is, Samira tells you that she thinks that she has picked up a bug that has been going around the hospital. She informs you that she has come into work because she does not want to miss the teaching session this afternoon. Samira has already missed several teaching sessions.
    Rank in order the importance of the following considerations in the management of this situation (1= Most important; 5= Least important).
    * Option A: The ability of Samira to meet her educational needs
    * Option B: That Samira could pass on the illness to patients or colleagues
    * Option C: That Samira should be resting in order to recover from the illness
    * Option D: Whether there are enough staff to see the patients should Samira be off sick
    * Option E: That Samira has already missed several teaching sessions this year
A

Correct Key: BCADE
Rationale: The most important consideration here would be your colleague’s ability to pass on the illness to her patients or colleagues (option B). There may be immunocompromised individuals in whom the illness may present in a potentially life-threatening form and therefore this is the most important consideration. Next would be option C as it is important that she look after health for her mental and physical wellbeing. Option A would be a consideration in the wider scheme of things but would not be an immediate priority. Options D is not an important consideration in determining if Samira can go off sick, as she is putting others at risk by attending the hospital when she is unwell. The least important consideration is option E, as there are likely to be more opportunities in the future for her to catch-up on the teaching sessions that she has missed.

66
Q
  1. You are working on a busy hospital ward. It is 16:30 and you are due to finish at 17:00 to attend a meeting with your educational supervisor. You have been asked by a registrar/specialty trainee to carry out a task before you leave, which is low priority and could be done tomorrow. The registrar/specialty trainee tells you that it is an easy task and it normally takes her 30 minutes. However, you completed the task once previously and it took much longer.
    Rank in order the appropriateness of the following actions in response to this situation (1= Most appropriate; 5= Least appropriate).
    * Option A: Inform the registrar/specialty trainee that you are unable to help
    * Option B: Explain that this task has taken you longer than 30 minutes previously, and you need to leave at 17:00 for a meeting
    * Option C: Agree to complete the task
    * Option D: Ask if you can help to find a different colleague to help with the task as you need to leave at 17:00 for a meeting
    * Option E: Ask the registrar/specialty trainee if she can find another colleague to complete the task
A

Correct Key: BDECA
Rationale: It is important that you do not miss your prearranged meeting with educational supervisor unless it is a medical emergency as it is important part of your training and also impacts your supervisor’s time. However, you still have 30 minutes before your shift ends so the most appropriate response will be to discuss your previous experience of this task taking more than 30 minutes and your prearranged educational supervision meeting (option B) with the registrar, as she may not be aware of it. Helping her to find an alternative solution (option D) is the next appropriate action and is proactive in finding a colleague to help with the task. Option E would be the next appropriate action, although this does not offer the registrar any support in finding support with the task, any she may be busy completing more urgent tasks. By agreeing to complete the task (option C) knowing that it is not possible to complete in 30 minutes is not appropriate, as your supervisor has allocated time and waiting for the meeting, which is important for your training requirement. Refusing to help (A) is unprofessional and is the least appropriate action.

67
Q
  1. You have had a busy day on-call* for the wards. The night shift FY1, Lucas, phones to say that he is going to be 20 minutes late for the shift handover. You are scheduled to meet your family after work as they have not seen you for a long time. Lucas tells you to write a list of tasks and to leave it on the nurses’ desk for him to review when he arrives.
    Rank in order the importance of the following considerations in the management of this situation (1= Most important; 5= Least important).
    * Option A: You are responsible for completing a thorough handover at the end of your shift
    * Option B: The inconvenience to your family if you are late
    * Option C: If there is another staff member available who could share the information with Lucas when he arrives
    * Option D: The level of detail required to effectively explain the shift handover information
    * Option E: It is not your responsibility to ensure that Lucas arrives on time for his shift
A

Correct Key: ADCEB
Rationale: Handover of information is a crucial step in continuity of patient care and patient safety. Leaving list of tasks on the nurse’s desk is not professional. Most important consideration is that it is your responsibility to complete the handover (A) as you have looked after the patients in your shift. It is also very important that the person handing over patient care not only has the in-depth knowledge about what needs to be handed over to the next team (D) but someone who can address any questions from the team taking over patient care. It may be that you are the only person who has this knowledge in which case you will need to wait until Lucas arrives. It is somewhat important in this situation to explore if another team member can complete an effective handover (C) which may allow you to leave as long you are confident that this will not compromise patient safety. At the time of dealing with the situation It is not important as to who is responsible for staff to arrive on time (E) or the inconvenience incurred to you and family (B) as it is expected that in an emergency you will prioritise patient safety over personal inconvenience.

68
Q
  1. You are working an on-call shift on a Friday evening. You are called to a ward to see the family of a patient who has just died. The patient and relatives are members of a religion that states burials should happen as soon as possible after death. You do not know the patient, although you are aware that your FY1 colleague, Claire, was treating the patient earlier today. As you enter the ward,
    the patient’s relatives immediately request that you contact Claire as they want
    Rank in order the appropriateness of the following actions in response to this situation (1= Most appropriate; 5= Least appropriate).
    * Option A: Telephone Claire to ask that she comes back to the hospital to sign the patient’s death certificate * Option B: Escalate the relatives’ request to the on-call consultant * Option C: Tell the relatives that Claire has left the hospital and will not be able to return to sign the death certificate until her next shift on Monday * Option D: Sign the patient’s death certificate * Option E: Contact the ward manager to see if there are any other doctors in the hospital who are able to sign the death certificate
A

Correct key: EBCAD
Rationale: This question is testing your ability to respond to a family request when the doctor that the family was previously in contact with is no longer on shift. (E) offers the quickest solution to the issue and allows you to continue with your on-call duties whilst an alternate doctor who is working is identified. (B) will provide appropriate senior input in real time, which will give advice as to the next steps for action. However, the on-call consultant may also not have seen the patient and may not advance the attempt to provide a prompt death certificate. (C) will disappoint the relatives but may in some circumstances be an appropriate response if no-one is in a position to issue the certificate. (A) is inappropriate in that Claire is not on duty at the hospital and may be on protected rest time. (D) is least appropriate, as you have never met the patient you should not complete their death certificate.

69
Q
  1. You have recently started a new rotation and your FY2 colleague is responsible for allocating the ward tasks between the two of you. After a couple of weeks, it appears to you that your FY2 colleague is allocating most of the work to you. You are struggling to finish all of your workload on time, while your FY2 colleague appears to have more free time and she does not offer to help complete your tasks.
    Rank in order the appropriateness of the following actions in response to this situation (1= Most appropriate; 5= Least appropriate).
    * Option A: Monitor the situation for a bit longer to see if it improves
    * Option B: Ask your FY2 colleague if she could allocate the ward tasks more equally between the two of you
    * Option C: Ask another colleague on the ward if they think your FY2 colleague is being unfair with allocating the ward tasks
    * Option D: Talk to your clinical supervisor* about the situation
    * Option E: Ask your FY2 colleague if there is a reason why she is allocating more of the ward tasks to you
A

Correct Key: EBDCA
Rationale: This is a situation which needs combination of professionalism and conflict resolution skills for satisfactory outcomes. Asking the concerned FY2 directly (E) is the most appropriate response because it uses the process of feedback to share your observation with her as she may not have done it deliberately and this may resolve the situation. (B) is also very appropriate and fair but may be seen simply as a demand rather than encouraging the colleague to reflect. Seeking help from your supervisor (D) is an appropriate action but you may wish to resolve the conflict within the team by a mutual discussion first but if the situation does not improve then this is certainly a very appropriate action. Asking other colleagues (C) may or may not be helpful as other colleagues may not agree with you but monitoring the situation (A) is least likely to resolve the situation and hence least appropriate.

70
Q
  1. You are working on a busy medical ward. The nurse in charge has asked you to complete a discharge summary urgently as a patient’s transport has already been booked. Whilst typing the summary, you are interrupted several times by colleagues who need your help, and you are finding it very difficult to concentrate on the summary. A nurse then approaches you and asks you to change a patient’s prescription for the drug round later that evening.
    Rank in order the appropriateness of the following responses to say in this situation. Assume they are all said in a polite way (1= Most appropriate; 5= Least appropriate).
    * Option A: “I already have a lot of other requests to deal with.”
    * Option B: “I’m completing an urgent discharge summary, I will change the prescription after.”
    * Option C: “I’m completing an urgent discharge summary, is anyone else able to help?”
    * Option D: “I’ll never get any tasks done if I keep getting interrupted.”
    * Option E: “I will come and find you after I have completed this task, to discuss your request.”
A

Correct Key: CBEAD
Rationale: In this scenario, you are clearly under a lot of pressure and have a lot of jobs to do all at the same time. The ward is very busy, and you have been swamped with requests from all directions. Your main concern is that you need to complete this discharge summary urgently, therefore this should be prioritised. The scenario establishes that the prescription that needs changing is for a drug round later in the evening, therefore is not an urgent job currently.
(C) would be the most appropriate as you are explaining why you cannot help immediately but have given a suggestion to have the job done by another colleague. Therefore, this is the best option. (B) similarly, also provides the nurse with your rationale, however, does not provide a solution for the job to be done quicker. (E) is not as appropriate as you have not communicated what it is that you are working on, therefore the nurse may think that you simply do want to help her out. You have also not given much reassurance about helping with the prescription change. (A) and (D) are not appropriate as you are simply dismissing the nurse’s request to have the prescription changed, which ultimately is putting the patient at risk. In addition, you are not offering an alternative solution to the nurse which is not helpful at all. (A) is slightly better than (D), the latter of which does not explain why you cannot help and sounds more like a moan or that you are blaming the nurse for stopping you from completing your tasks.

71
Q
  1. You are just starting a night shift for the hospital surgical team. You are asked to review a patient that the previous FY1 has just seen. You begin to read the patient file written by the previous FY1. The final section in the patient file is titled patient management plan, however, you are struggling to read this section as the handwriting becomes very messy. You assume that this is because the FY1 rushed the last section of the patient file before leaving.
    Rank in order the importance of the following considerations in the management of this situation (1= Most important; 5= Least important).
    * Option A: How the FY1 will handle receiving the feedback about his handwriting
    * Option B: Whether the information provided in the patient file is understood
    * Option C: The reason why the FY1 had to rush the last section of the patient file
    * Option D: Your need to start completing your tasks
    * Option E: Other ways you could find out about patient management plan
A

Correct key: BEDCA
Rationale: You have been asked to review a patient therefore, there is a clinical need to assess this patient. Given that the FY1 has also just seen him/her, but the nursing team have already re-bleeped you, there may have been a deterioration and the patient might be very unwell. Regardless, your priority lies with the patient. In order to make an accurate assessment of the patient, you need a good understanding and timeline as to what has happened to the patient previously, including diagnosis, treatment and management plan (therefore (B) is most important). As the scenario has started that you are struggling to read the FY1’s documentation, you should seek other ways to find out about the patient’s management plan, in order to fill in the gaps of what you do not know so that you can make a safe and accurate assessment (E). Your need to start completing your tasks (D) is somewhat important, as you may have other jobs to do or patients to review. However, this should not be prioritised over patient safety. Without correct information on the patient, blindly “completing your task” would be unsafe and may put the patient at risk of harm.
Receiving feedback and constructive criticism, then being able to reflect and improve on this is a key skill that a doctor needs to have. Only through this way can the doctor improve themselves and learn from their mistakes. It is stated in the GMC Good Medical Practice that a doctor’s recording keeping needs to be clear and handwriting needs to be legible. Regardless of the reason for the messy handwriting, if the FY1s handwriting is causing problems regarding communication and patient care, then they need to be made aware and work on it to ensure patient safety (C and A).

72
Q
  1. You have been completing an audit with another FY1 doctor, Zara. You are near the end of your rotation and have only collected half of the data; there are two wards remaining to collect data from. You and Zara have both observed that a general trend has emerged in the data that you have collected so far. One morning, Zara informs you that she has fabricated the rest of the data, based on the trend that had emerged in the existing data, and that she has submitted the report to your joint clinical supervisor*, Dr Hadi. The following week, Dr Hadi informs you that she is very pleased with the audit and wants both you and Zara to attend an international conference to present the findings.
    Rank in order the appropriateness of the following actions in response to this situation (1= Most appropriate; 5= Least appropriate).
    * Option A: Inform Dr Hadi that the data was fabricated by Zara
    * Option B: Tell Dr Hadi that you did not have any input into the audit so Zara should attend the conference alone
    * Option C: Tell Dr Hadi that there was a mistake with the data, so you and Zara will need to collect some more before presenting at the conference
    * Option D: Attend the conference with Zara, but refuse to present any of the findings
    * Option E: Suggest to Zara that she admits to Dr Hadi that she fabricated the data
A

Correct key: EACBD
Rationale: This question tests your probity. (E) addresses the probity issue and is the most appropriate response as it allows the individual responsible to take ownership and responsibility for their behaviour. (A) addresses the probity issue but doesn’t create the opportunity for Zara to admit her mistakes which is the preferred option. If she won’t inform Dr Hadi it is the most appropriate next action. (C) ensures that you aren’t involved in delivering fraudulent data but doesn’t address the key issue. (B) ensures that you aren’t involved but directly permits fraudulent data to be presented, which potentially creates a fitness to practise issue for you and Zara. (D) doesn’t address any of the issues and not taking any action at all clearly implicates you in the delivery of fraudulent data. This is the least appropriate response.

73
Q
  1. You are working in the Emergency Department and have just reviewed a 10 year old boy with a probable diagnosis of appendicitis. You attempt to contact the surgical registrar/specialty trainee* several times, and when he responds, he is very rude and states that “just like all the other FY1 doctors, you are unable to use your knowledge to make the right decisions”. You have never had an interaction with him before, but your FY1 colleagues have said that he can be very rude.
    Rank in order the appropriateness of the following responses to say in this situation. Assume they are all said in a polite way (1= Most appropriate; 5= Least appropriate). * Option A: “I am sorry to disturb you, but I am worried about this patient” * Option B: “Can you explain why you feel that I have made a wrong decision?” * Option C: “Would you like me to explain why I bleeped you?” * Option D: “I feel that you are being very rude” * Option E: “My FY1 colleagues have said that you are very rude”
A

Correct key: ACBDE
Rationale: Incivility amongst colleagues can be difficult to manage and upsetting to be on the receiving end of. However, the most important thing in the short-term is the safety of the patient. Unprofessional behaviour needs be dealt with, but this can be done later and separately if needed. It is important to diffuse any potential arguments and keep the conversation professional and patient centred. (A) is the most appropriate thing to say as it is polite, professional, but gets to the point that you are worried about this patient, who is the priority in this situation. (C) is an appropriate response, as it is polite and reiterates that there is a purpose to you bleeping him regarding patient care and that you need to explain this. (B) is a somewhat appropriate response. Although it is opening the dialogue, it does not appear from the information that we have been given, that the wrong decision has been made, therefore inviting self-criticism is unlikely to be helpful for the patient or help building relationships going forward. (D) is not an appropriate response as although you think that he has been rude, telling him in this moment is unlikely to move the conversation forward and has lost track of the patient focus. (E) is the least appropriate thing to say, as it implies that people are gossiping behind his back and is likely to escalate tensions between you and the surgical registrar/specialty trainee. This response has also lost sight of patient care.

74
Q
  1. You are on your break and you overhear a fellow FY1 colleague in the canteen speaking about one of her patients that is currently on the ward. Your FY1 colleague is talking loudly, so you can clearly hear what she is saying from a distance. You look around and notice that there are also relatives and other patients currently in the canteen. You approach your FY1 colleague and try to make her aware of the issue, but you are dismissed with a wave before being able to say anything.
    Rank in order the importance of the following considerations in the management of this situation (1= Most important; 5= Least important).
    * Option A: That you have already attempted to say something to your FY1 colleague, and she has dismissed you
    * Option B: That the relatives of the patient that is being spoken about may be in the canteen
    * Option C: That there are lots of other members of medical staff that could make your FY1 colleague aware of the issue
    * Option D: That your FY1 colleague is breaching patient confidentiality
    * Option E: That patients and relatives in the canteen may not know that your FY1 colleague is speaking about a patient
A

Correct Key: DBACE
Rationale: This scenario demonstrates a huge breach in patient confidentiality, which is paramount within medical ethics. This needs to be taken extremely seriously and needs to be addressed immediately, as it could lead to a breakdown in the patient-doctor relationship and trust towards healthcare professionals, which underpins all the work that we do. Therefore, (D) is the most important consideration at this stage.
The breach in confidentiality is occurring in a public space (the canteen) where there are lots of people, including relatives and other patients. These relatives may be of the patient who is being discussed (B). If so, the actions of your FY1 colleague could cause them a lot of distress and upset them. Furthermore, if the relatives inform the patient in the scenario, and subsequently patient-doctor relationship is broken, this may compromise the level of care that can be delivered to the patient and cause harm.
Even though your FY1 colleague has seemingly dismissed you already (A), this is not a top consideration as it is still your duty to make her aware of the problem, despite how many attempts it takes. It is everyone’s duty to maintain confidentiality and if you have witnessed that it has been breached, then you need to take action yourself. For this same reason, you cannot simply rely on other medical staff to make her aware of the issue, which is why option C is ranked lower than A.
Regardless of whether others can identify the patient through what your FY1 colleague is saying (E), your FY1 colleague should know that certain places, especially public areas, are not suitable for discussing patient matter, and this should only be done in a space that is deemed to be safe and only with relevant people listening.

75
Q
  1. You are working on a busy ward. You have made three unsuccessful attempts to insert a cannula into a patient who requires antibiotics for a wound infection. You do not believe that you will be successful if you make any further attempts. The patient is becoming agitated and is losing patience with you. You call a registrar/specialty trainee* for assistance, but he informs you that someone of your level should be able to do this, and that it is not his responsibility as he has other things to do.
    Rank in order the appropriateness of the following actions in response to this situation (1= Most appropriate; 5= Least appropriate).
    * Option A: Attempt to insert the cannula again
    * Option B: Leave it in the evening handover for a colleague to complete
    * Option C: Call the registrar/specialty trainee again and ask him to reconsider his decision
    * Option D: Ask another colleague to assist you
    * Option E: Ask a colleague for advice on how to proceed
A

Correct key: DECBA
Rationale: In this scenario, you have done the correct thing by recognising your limits and escalating to a senior for support and advice. Unfortunately, the registrar has not taken it well and is currently too preoccupied with other tasks to help you. In this case, it would be most appropriate to ask another colleague to assist you (D). Other colleagues may be less busy than the registrar, and may have more experience in cannulation than you, therefore would be able to offer you a helping hand. Alternatively, you could ask a colleague for advice on how to proceed (E), which is reasonable as they have likely encountered similar situations in the past. However, it is not as appropriate as option D whereby you are asking for their assistance, which is more straight forward and gets the message across that you need their help now. You could consider calling the registrar again (C) as ultimately, the patient needs a cannulation to receive his medication. However, this is not the most appropriate as you have already spoken to him once and he sounded relatively reluctant to help, so it would be best to seek assistance from others.
Option A is the least appropriate action because you have already had three unsuccessful attempts and the patient is becoming agitated and losing patience with you. Furthermore, you are not confident that you will be successful with any further attempts. If you are unsuccessful again, the patient may be too frustrated to allow anyone to cannulate him and therefore will cause more harm as the patient will not be able to receive his intravenous antibiotics. Although option B also delays the administration of medication, is it more appropriate than option A, which may jeopardise any chance of cannulating the patient.